Как найти промежуток убывания функции без графика

Свойства функции. Возрастание и убывание, наибольшее и наименьшее значения, нули, промежутки знакопостоянства.

теория по математике 📈 функции

Каждый из нас встречался с разными графиками, как на уроках, так и в жизни. Например, рассматривали, как изменяется температура воздуха в определенный период времени.

На рисунке видно, что температура воздуха была отрицательной с 0 часов до 6 часов, а также с 20 до 24 часов. Еще можем сказать, что температура повышалась до 14 часов, а затем понижалась. То есть по данному графику мы смогли определить некоторые свойства зависимости температуры воздуха от времени суток.

Остановимся подробнее на свойствах функций.

Нули функции

Нули функции – это значение аргумента, при которых функция обращается в нуль. Если смотреть нули функции на графике, то берем точки, где график пересекает ось х.

На рисунке он пересекает ось х при х=-1; х=4; х=6. Эти точки пересечения выделены красным цветом. Внимание!

Существует функция, которая не будет иметь нули функции. Это гипербола. Вспомним, что функция имеет

Вид — группа особей, сходных по морфолого-анатомическим, физиолого-экологическим, биохимическим и генетическим признакам, занимающих естественный ареал, способных свободно скрещиваться между собой и давать плодовитое потомство.

График функции у=k/x выглядит следующим образом: По данному рисунку видно, что нулей функции не существует. Как найти нули функции?

  1. Для того чтобы найти нули функции, которая задана формулой, надо подставить вместо у число нуль и решить полученное уравнение.
  2. Если график функции дан на рисунке, то ищем точки пересечения графика с осью х.

Рассмотрим примеры нахождения нулей функции. Пример №1. Найти нули функции (если они существуют):

а) Для нахождения нулей функции необходимо в данную формулу вместо у подставить число 0, так как координаты точки пересечения графика с осью х (х;0). Нам нужно найти значение х. Получаем 0 = –11х +12. Решаем уравнение. Переносим слагаемое, содержащее переменную, в левую часть, меняя знак на противоположный: 11х=22

Находим х, разделив 22 на 11: х=22:11

Таким образом, мы нашли нуль функции: х=2

б) Аналогично во втором случае. Подставляем вместо у число 0 и решаем уравнение вида 0=(х + 76)(х – 95). Вспомним, что произведение двух множителей равно 0 тогда и только тогда, когда хотя бы один из множителей равен 0. Таким образом, так как у нас два множителя, составляем два уравнения: х + 76 = 0 и х – 95 = 0. Решаем каждое, перенося числа 76 и -95 в правую часть, меняя знаки на противоположные. Получаем х = – 76 и х = 95. Значит, нули функции это числа (-76) и 95.

в) в третьем случае: если вместо у подставить 0, то получится 0 = – 46/х, где для нахождения значения х нужно будет -46 разделить на нуль, что сделать невозможно. Значит, нулей функции в этом случае нет.

Пример №2. Найти нули функции у=f(x) по заданному графику.

Находим точки пересечения графика с осью х и выписываем значения х в этих точках. Это (-4,9); (-1,2); 2,2 и 5,7. У нас на рисунке точки пересечения выделены красным цветом.

Промежутки знакопостоянства

Промежутки, где функция сохраняет знак (то есть значение y либо положительное на этом промежутке, либо отрицательное), называется промежутками знакопостоянства.

Рассмотрим по нашему рисунку, на какие промежутки разбивается область определения данной функции [-3; 7] ее нулями. По графику видно, что это 4 промежутка: [-3; -1), (-1;4), (4; 6) и (6; 7]. Помним, что значения из области определения смотрим по оси х.

На рисунке синим цветом выделены части графика в промежутках [-3; -1) и (4; 6), которые расположены ниже оси х. Зеленым цветом выделены части графика в промежутках (-1;4) и (6; 7], которые расположены выше оси х.

Значит, что в промежутках [-3; -1) и (4; 6) функция принимает отрицательные значения, а в промежутках (-1;4) и (6; 7] она принимает положительные значения. Это и есть промежутки знакопостоянства.

Пример №3. Найдем промежутки знакопостоянства по заданному на промежутке [-2; 10] графику функции у=f(x).

Функция принимает положительные значения в промежутках [-2; -1) и (3; 8). Обратите внимание, что эти части на рисунке выделены зеленым цветом.

Функция принимает отрицательные значения в промежутках (-1; 3) и (8; 10]. Обратите внимание на линии синего цвета.

Возрастание и убывание функции

Значения функции могут уменьшаться или увеличиваться. Это зависит от того, как изменяются значения х. Рассмотрим это свойство по рисунку.

На графике видно, что с увеличением значения х от -3 до 2 значения у тоже увеличиваются. Также с увеличением значения х от 5 до 7 значения у опять увеличиваются. Проще говоря, слева направо график идет вверх (синие линии). То есть в промежутках [-3; 2] и [5; 7] функция у=f(x) является возрастающей.

Посмотрим на значения х, которые увеличиваются от 2 до 5. В этом случае значения у уменьшаются. На графике эта часть выделена зеленым цветом. Слева направо эта часть графика идет вниз. То есть в промежутке [2;5] функция у=f(x) является убывающей.

Функция называется возрастающей в некотором промежутке, если большему значению аргумента из этого промежутка соответствует большее значение функции; функция называется убывающей в некотором промежутке, если большему значению аргумента из этого промежутка соответствует меньшее значение функции.

Свойства функций: чётность, промежутки знакопостоянства, монотонность.

Обращаем Ваше внимание, что в соответствии с Федеральным законом N 273-ФЗ «Об образовании в Российской Федерации» в организациях, осуществляющих образовательную деятельность, организовывается обучение и воспитание обучающихся с ОВЗ как совместно с другими обучающимися, так и в отдельных классах или группах.

Рабочие листы и материалы для учителей и воспитателей

Более 300 дидактических материалов для школьного и домашнего обучения

Итак, мы познакомились с функцией, узнали, что такое область определения и область значений функции. Теперь рассмотрим свойства функций. Их существует много, однако, изучаются они постепенно. В 9 классе мы знакомимся с нулями функции, промежутками возрастания и убывания (монотонность) и промежутками знакопостоянства и чётностью (нечётностью) функции. Рассмотрим их подробно.

Нулями функции называются значения независимой переменной (аргумента), при которых значение функции равно нулю. В графической интерпретации нулями функции являются абсциссы точек пересечения графика с осью абсцисс (осью х).

На графике нули функции: .

Для того, чтобы найти нули функции, заданной аналитически, необходимо решить уравнение: . Корни этого уравнения являются нулями функции.

Например, найти нули функции .

Промежутками знакопостоянства функции называются промежутки значений аргумента, на которых значения функции либо только положительны, либо только отрицательны. Другими словами, это те промежутки, на которых функция сохраняет свой знак.

Рассматривая график сверху, найдём промежутки знакопостоянства.

функция принимает только положительные значения на тех участках графика, где он находится выше оси Ох, т.е. при ;

функция принимает только отрицательные значения на тех участках графика, где он находится ниже оси Ох, т.е. при .

Для того, чтобы найти промежутки знакопостоянства функции, заданной аналитически, необходимо решить неравенства: и . Решения этих неравенств и будут промежутками знакопостоянства функции.

Например, найти промежутки знакопостоянства функции .

Это неравенство можно решить двумя способами: с помощью систем неравенств и методом интервалов. Метод интервалов будет рассмотрен нами чуть позже, поэтому воспользуемся системами неравенств. Произведение двух множителей положительно, если эти множители имеют одинаковый знак. Значит, получается совокупность двух систем:

Теперь находим промежутки, на которых функция принимает отрицательные значения.

Произведение двух множителей отрицательно, если эти множители имеют разные знаки, т.е.

Чётной называется функция, если противоположным значениям аргумента соответствуют одинаковые значения функции, т.е. . График чётной функции симметричен относительно оси ординат (оси Оу).

Нечётной называется функция, если противоположным значениям аргумента соответствуют противоположные значения функции, т.е. . График нечётной функции симметричен относительно начала координат.

На рисунке слева график чётной функции, на рисунке справа – нечётной функции.

Для того, чтобы определить чётность функции, заданной аналитически, необходимо в заданную функцию вместо х подставить –х и произвести упрощение. Если в результате получится функция, равная заданной, то функция чётная; если получится функция, противоположная заданной, то она нечётная; если не получится ни один из предложенных вариантов, то функция не является ни чётной, ни нечётной.

Например, исследовать на чётность функцию .

Находим значение этой функции при противоположном значении х, т.е.

Полученное выражение не совпадает с заданным и не противоположно ему, значит, функция не является ни чётной, ни нечётной. Её график не симметричен относительно оси Оу и не симметричен относительно начала координат.

Приведём ещё один пример: .

После упрощения получили выражение, полностью совпадающее с заданным. Значит, функция является чётной и её график симметричен относительно начала координат.

Функция называется возрастающей на некотором промежутке, если большему значению аргумента из этого промежутка соответствует большее значение функции (или меньшему значению аргумента соответствует меньшее значение функции), т.е. если при , то функция возрастающая.

Функция называется убывающей на некотором промежутке, если большему значению аргумента из этого промежутка соответствует меньшее значение функции (или меньшему значению аргумента соответствует большее значение функции), т.е. если при , то функция убывающая.

Для примера рассмотрим графики на рисунках выше.

Синий график: функция возрастает при

функция убывает при

Зелёный график: функция возрастает при

функция убывает при

Промежутки возрастания и убывания функции называются промежутками монотонности функции.

Если функция задана аналитически, то нахождение промежутков монотонности является более сложным процессом и он изучается в 11 классе. Мы ограничимся определением этих промежутков по графикам.

Наибольшим значением функции называется самое большое значение функции по сравнению со всеми остальными.

Наименьшим значением функции называется самое маленькое значение функции по сравнению со всеми остальными.

Строгое определение наибольшего и наименьшего значения функции будет дано в старших классах.

На синем графике наибольшего значения нет, т.к. график бесконечен в положительном направлении оси Оу. А наименьшее значение равно . Записывается это так: .

На зелёном графике нет ни наибольшего, ни наименьшего значения функции.

На рисунках изображены части графиков нечётных функций. Достройте эти графики.

Интервалы возрастания и убывания функции

Исследование функции с помощью производной

Определение : Точка х0 называется точкой локального минимума, если для любого х из окрестности точки х0 выполняется неравенство: f(x0) .
Точки минимума и максимума функции называются точками экстремума данной функции, а значения функции в этих точках – экстремумами функции.
Точками экстремума могут служить только критические точки I рода, т.е. точки, принадлежащие области определения функции, в которых производная f′(x) обращается в нуль или терпит разрыв.

Правило нахождения экстремумов функции y=f(x) с помощью первой производной

  1. Найти производную функции f′(x) .
  2. Найти критические точки по первой производной, т.е. точки, в которых производная обращается в нуль или терпит разрыв.
  3. Исследовать знак первой производной в промежутках, на которые найденные критические точки делят область определения функции f(x) . Если на промежутке f′(x) , то на этом промежутке функция убывает; если на промежутке f′(x)>0 , то на этом промежутке функция возрастает.
  4. Если в окрестности критической точки f′(x) меняет знак с «+» на «-», то эта точка является точкой максимума, если с «-» на «+», то точкой минимума.
  5. Вычислить значения функции в точках минимума и максимума.

С помощью приведенного алгоритма можно найти не только экстремумы функции, но и промежутки возрастания и убывания функции.

Пример №1 : Найти промежутки монотонности и экстремумы функции: f(x)=x 3 –3x 2 .
Решение: Найдем первую производную функции f′(x)=3x 2 –6x.
Найдем критические точки по первой производной, решив уравнение 3x 2 –6x=0; 3x(x-2)=0 ;x = 0, x = 2

Исследуем поведение первой производной в критических точках и на промежутках между ними.

x (-∞, 0) 0 (0, 2) 2 (2, +∞)
f′(x) + 0 0 +
f(x) возрастает max убывает min возрастает

f(0) = 0 3 – 3*0 2 = 0
f(2) = 2 3 – 3*2 2 = -4
Ответ: Функция возрастает при x∈(-∞ ; 0)∪(2; +∞); функция убывает при x∈(0;2);
точка минимума функции (2;-4); точка максимума функции (0;0).

Правило нахождения экстремумов функции y=f(x) с помощью второй производной

  1. Найти производную f′(x) .
  2. Найти стационарные точки данной функции, т.е. точки, в которых f′(x)=0 .
  3. Найти вторую производную f″(x) .
  4. Исследовать знак второй производной в каждой из стационарных точек. Если при этом вторая производная окажется отрицательной, то функция в такой точке имеет максимум, а если положительной, то – минимум. Если же вторая производная равна нулю, то экстремум функции надо искать с помощью первой производной.
  5. Вычислить значения функции в точках экстремума.

Отсюда следует, что дважды дифференцируемая функция f(x) выпукла на отрезке [a, b], если вторая производная f»(x) ≥ 0 при всех х [a, b].

Все вычисления можно проделать в онлайн режиме.

Пример №2 . Исследовать на экстремум с помощью второй производной функцию: f(x) = x 2 – 2x — 3.
Решение: Находим производную: f′(x) = 2x — 2.
Решая уравнение f′(x) = 0, получим стационарную точку х =1. Найдем теперь вторую производную: f″(x) = 2.
Так как вторая производная в стационарной точке положительна, f″(1) = 2 > 0, то при x = 1 функция имеет минимум: fmin = f(1) = -4.
Ответ: Точка минимума имеет координаты (1; -4).

источники:

http://infourok.ru/svoystva-funkciy-chyotnost-promezhutki-znakopostoyanstva-monotonnost-3866952.html

http://math.semestr.ru/math/intervals.php

Как найти промежутки возрастания и убывания функции

Определение промежутков возрастания и убывания функции – это один из основных аспектов исследования поведения функции наряду с нахождением точек экстремумов, в которых происходит перелом от убывания к возрастанию и наоборот.

Как найти промежутки возрастания и убывания функции

Инструкция

Функция y = F(x) является возрастающей на определенном интервале, если для любых точек x1 F(x2), где x1 всегда > x2 для любых точек на интервале.

Существуют достаточные признаки возрастания и убывания функции, которые вытекают из результата вычисления производной. Если производная функции положительна для любой точки интервала, то функция возрастает, если отрицательна – убывает.

Чтобы найти промежутки возрастания и убывания функции, нужно найти область ее определения, вычислить производную, решить неравенства вида F’(x) > 0 и F’(x)

Рассмотрим пример.
Найти промежутки возрастания и убывания функции для y = (3·x² + 2·x — 4)/x².

Решение.
1. Найдем область определения функции. Очевидно, что выражение, стоящее в знаменателе, должно всегда быть отличным от нуля. Поэтому точка 0 исключается из области определения: функция определена при x ∈ (-∞; 0)∪(0; +∞).

2. Вычислим производную функции:
y’(x) = ((3·x² + 2·x — 4)’ ·x² – (3·x² + 2·x — 4) · (x²)’)/x^4 = ((6·x + 2) ·x² – (3·x² + 2·x — 4) ·2·x)/x^4 = (6·x³ + 2·x² – 6·x³ – 4·x² + 8·x)/x^4 = (8·x – 2·x²)/x^4 = 2· (4 — x)/x³.

3. Решим неравенства y’ > 0 и y’ 0;
(4 — x)/x³

4. Левая часть неравенства имеет один действительный корень х = 4 и обращается в бесконечность при x = 0. Поэтому значение x = 4 включается и в промежуток возрастания функции, и в промежуток убывания, а точка 0 не включается никуда.
Итак, искомая функция возрастает на промежутке x ∈ (-∞; 0) ∪ [2; +∞) и убывает при x (0; 2].

Рассмотрим пример.
Найти промежутки возрастания и убывания функции для y = (3·x² + 2·x — 4)/x².

Решение.
1. Найдем область определения функции. Очевидно, что выражение, стоящее в знаменателе, должно всегда быть отличным от нуля. Поэтому точка 0 исключается из области определения: функция определена при x ∈ (-∞; 0)∪(0; +∞).

2. Вычислим производную функции:
y’(x) = ((3·x² + 2·x — 4)’ ·x² – (3·x² + 2·x — 4) · (x²)’)/x^4 = ((6·x + 2) ·x² – (3·x² + 2·x — 4) ·2·x)/x^4 = (6·x³ + 2·x² – 6·x³ – 4·x² + 8·x)/x^4 = (8·x – 2·x²)/x^4 = 2· (4 — x)/x³.

3. Решим неравенства y’ > 0 и y’ 0;
(4 — x)/x³

4. Левая часть неравенства имеет один действительный корень х = 4 и обращается в бесконечность при x = 0. Поэтому значение x = 4 включается и в промежуток возрастания функции, и в промежуток убывания, а точка 0 не включается никуда.
Итак, искомая функция возрастает на промежутке x ∈ (-∞; 0) ∪ [2; +∞) и убывает при x (0; 2].

4. Левая часть неравенства имеет один действительный корень х = 4 и обращается в бесконечность при x = 0. Поэтому значение x = 4 включается и в промежуток возрастания функции, и в промежуток убывания, а точка 0 не включается никуда.
Итак, искомая функция возрастает на промежутке x ∈ (-∞; 0) ∪ [2; +∞) и убывает при x (0; 2].

Источники:

  • как найти на функции промежутки убывания

Войти на сайт

или

Забыли пароль?
Еще не зарегистрированы?

This site is protected by reCAPTCHA and the Google Privacy Policy and Terms of Service apply.

Что такое возрастание функции

В начале прочитаем определение возрастания функции.

Запомните!
!

Функция « y(x) » называется возрастающей на некотором промежутке, если

для любых
« x1 » и « x2 »
принадлежащих данному промежутку, таких, что « x2 > x1 »
выполняется неравенство

« y( x2 ) > y( x1 )».

Определение сложно понять без наглядного примера.
Поэтому сразу перейдём к разбору задачи на возрастание функции.

По-другому можно сказать, что, если каждому бóльшему значению « x »
соответствует бóльшее значение « y », значит,
функция « y(x) » возрастает.

x2 > x1
y( x2 ) > y( x1 )

Обязательное условие возрастания функции

Давайте разберем определение возрастания функции на конкретном примере.

Разбор примера

Возрастающей или убывающей является функция « y = 9x − 4 » ?

Для начала определим
область определения функции
« y = 9x − 4 ».

y = 9x − 4
D(y): x ∈ R
,
то есть « x » —
любое действительное число.

Построим график функции
« y = 9x − 4 ».
Так как функция
« y = 9x − 4 »
линейная, ее график — прямая.

Используем правила построения графика линейной функции. Нам достаточно найти две точки, чтобы построить ее график.

Область определения функции
« y = 9x − 4 » — все действительные числа,
поэтому можно подставить любое число вместо « x » и вычислить « y » по
формуле функции
« y = 9x − 4 ». Например, возьмем
« x = 0 ».

x = 0
y(x) = 9x − 4
y(0) = 9 · 0 − 4 = −4

Для второй точки возьмем « x = 1 ».

x = 1
y(x) = 9x − 4
y(1) = 9 · 1 − 4 = 5

Отметим две полученные
точки «(0; −4)» и «(1; 5)» на

координатной плоскости
и проведем через них прямую.

график линейной функции y = 9x - 4

Докажем, что функция
« y = 9x − 4 » возрастает на всей своей области определения двумя способами: по ее графику и
аналитически
(по ее формуле).

Как определить по графику, что функция возрастает

По определению возрастания функции мы знаем, что
если « x » увеличивается,
то « y » тоже должен увеличиваться.

На рисунке ниже видно, что график функции « y = 9x − 4 »
«идет в гору». Другими словами, при увеличении « x »
растет
значение « y » .

график линейной функции возрастает

В этом можно убедиться, если взять две любые точки на графике. Например, точки, по
которым мы построили график функции. Назовем эти точки:
« (·)A » и « (·)B ».

точки А и В на графике

У первой точки « (·)A »
координаты:
x1 = 0 ;   y1 = − 4

У второй точки « (·)B » координаты:
x2 = 1 ;   y2 = 5

На примере точек « (·)A » и « (·)B » видно, что
при увеличении
« x ( x2 > x1 )»
растет
« y ( y2 > y1 ) ».
Поэтому график зрительно «идет в гору».

Как по формуле доказать, что функция возрастает

Вернёмся к нашей функции
« y = 9x − 4 ».

По графику мы поняли, что
функция « y = 9x − 4 » возрастает,
так как ее график «идет в гору».
Но как доказать по формуле, что функция
возрастает на всей своей области определения?

Запомните!
!

Функция возрастает на всей области определения, когда при
« x2 > x1 »
выполняется условие
« y( x2 ) > y( x1 ) ».

Формулировка выше не самая простая для понимания. Давайте разберем ее на практике.

По определению возрастания функции нам нужно доказать, что при
« x2 > x1 » увеличивается значение функции
« y( x2 ) > y( x1 ) ».

Но как нам найти значения функции
« y( x1 )» и
«y( x2 ) »?

Для нахождения « y( x1 )» и
«y( x2 ) »

достаточно подставить « x1 » и
« x2 » в исходную формулу « y = 9x − 4 ».

y( x1 ) = 9x1 − 4
y( x2 ) = 9x2 − 4

Теперь запишем обязательное условие возрастания функции.

x2 > x1
y( x2 ) > y( x1 )

Обязательное условие возрастания функции

Подставим в неравенство
« y( x2 ) >
y( x1 ) » полученные формулы

« y( x1 ) = 9x1 − 4» и
« y( x2 ) = 9x2 − 4 » .

y( x2 ) > y( x1 )
9x2 − 4 > 9x1 − 4

Упростим полученное
неравенство.

9x2 − 9x1 > − 4 + 4
9x2 − 9x1 > 0

Вынесем общий множитель
в левой части неравенства.

9(x2 − x1) > 0

Разделим левую и правую часть на «9».

При делении нуля на любое число получается ноль.

x2 − x1 > 0
x2 > x1

Мы доказали, что выполняется исходное условие возрастания функции «x2 > x1».
Отсюда следует, что функция
« y = 9x − 4 » возрастает на всей области определения.

В завершении вместо ответа следует написать фразу:
«Что и требовалось доказать».


Посмотрим другой пример, где требуется доказать, что функция возрастает.

Разбор примера

Доказать, что функция возрастает на всей области определения: y = 13x − 1

По аналогии с предыдущим примером составим неравенства, которые доказывают, что функция возрастает.

x2 > x1
y( x2 ) > y( x1 )

Обязательное условие возрастания функции

Вместо « y( x1 )» и
«y( x2 ) » запишем
формулу функции « y = 13x − 1 » и упростим полученное неравенство.

y( x2 ) > y( x1 )

13x2 − 1 > 13x1 − 1

13x2 − 13x1 > 1 − 1

13(x2 − x1) > 0 |: 13

>

x2 − x1 > 0

x2 > x1

Что и требовалось доказать.

Что такое убывание функции

Запомните!
!

Функция « y(x) » называется убывающей на некотором промежутке, если для любых
« x1 » и « x2 »
принадлежащих данному промежутку, таких,
что « x2 > x1 »
выполняется неравенство « y( x2 ) < y( x1 )».

x2 > x1
y( x2 ) < y( x1 )

Обязательное условие убывания функции

Как по графику понять, что функция убывает

Разбор примера

Доказать, что функция убывает на всей области определения: y = 1 − 3x

По определению убывания функции мы знаем, что,
если « x »
растет, то
« y » должен уменьшаться.

Построим график функции
« y = 1 − 3x ». Ее график — прямая, поэтому нам будет достаточно двух точек.

Область определения функции
« y = 1 − 3x » — все действительные числа,
поэтому можно поставить любое число вместо « x » и вычислить « у » по
формуле функции
« y = 1 − 3x ». Например, возьмем
« x = 0 »
и « x = 1 ».

x = 0
y(x) = 1 − 3x
y(0) = 1 − 3 · 0 = 1

(·) А (0; 1)

x = 1
y(1) = 1 − 3x
y(1) = 1 − 3 · 1 = 1 − 3 = −2

(·) B (1; −2)

Построим график функции
« y = 1 − 3x » по полученным точкам
« (·)A » и « (·)B ».

график линейной функции y = 1 - 3x

На графике функции видно, что зрительно график «спускается с горы», то есть функция убывает. Другими словами, при увеличении
« x »
уменьшается
значение
« y » .

Как по формуле доказать, что функция убывает

Вернёмся к нашей функции
« y = 1 − 3x ».

По ее графику мы поняли, что функция убывает, так как график «спускается с горы». Но как доказать по формуле,
что функция « y = 1 − 3x » убывает на всей области определения?

Запомните!
!

Чтобы доказать, что функция убывает требуется доказать, что при любых
« x2 > x1 » выполняется

« y( x2 ) < y( x1 ) ».

Давайте разберем на примере функции
« y = 1 − 3x ». Докажем, что она убывает
на всей своей области определения.

x2 > x1
y( x2 ) < y( x1 )

Обязательное условие убывания функции

Подставим « y( x1 )» и
«y( x2 ) » в
формулу функции « y = 1 − 3x » и упростим полученное неравенство.

y( x2 ) < y( x1 )

1 − 3x2 < 1 − 3x1

3x1 − 3x2 < 1 − 1

3(x1 − x2) < 0 | :3

<

x1 − x2 < 0

−x2 < −x1

Умножим на « −1 » левую и правую часть неравенства. При
умножении неравенства на отрицательное число знак неравенства поменяется на
противоположный.

−x2 < −x1 | · (−1)

x2 > x1

Что и требовалось доказать.

Как по графику функции определить
возрастание и убывание

Потренируемся только по графику функции определять промежутки возрастания и убывания функции.

Разбор примера

На рисунке ниже изображён график функции, определенной на множестве действительных чисел.
Используя график, найдите промежутки возрастания и промежутки убывания функции.

Как по графику функции определить возрастает или убывает функция

Отметим с помощью штриховых линий промежутки, где график функции убывает
(«спускается с горы») и где он возрастает («идет в гору»).

промежутки возрастания и убывания функции

Запишем через знаки неравенств,
какие значения принимает « x » на полученных промежутках.
Обратите внимание, что во всех случаях при указании промежутков, мы указываем, что их
концы входят в промежуток, то есть используем знаки нестрогого неравенства.

промежутки возрастания и убывания функции через неравенства

Остаётся записать полученные промежутки возрастания и убывания функции в ответ.

Ответ:

  • функция убывает при
       x ≤ −2;     0 ≤ x ≤ 3,5
  • функция возрастает при
        −2 ≤ x ≤ 0 ;     x ≥ 3,5

Более грамотно будет записать ответ с помощью специальных
математических символов.

Ответ:

  • функция убывает на промежутках    
    x ∈ (−∞ ; −2] ∪ [0; 3,5]
  • функция возрастает на промежутках     x ∈ [−2 ; 0] ∪ [3,5 ; +∞]

При каких значениях
« m »
функция является убывающей или возрастающей

Ещё один тип заданий, в которых требуется определить,
при каких
« m » ( « а, b » или других буквах) функция убывает или возрастает.

Разбор примера

При каких значениях « m » функция

« y = mx − m − 3 + 2x » является убывающей?

Обратимся снова к определению убывания функции. Вспомним, как записать условия убывания функции с точки зрения формул.

x2 > x1
y( x2 ) < y( x1 )

Обязательное условие убывания функции

Запишем эти условия, используя формулу функции « y = mx − m − 3 + 2x », заданную в
задаче. Вместо
« x »
подставим « x1 » и « x2 ».

y( x2 ) < y( x1 )

mx2 − m − 3 + 2x2 < mx1 − m − 3 + 2x1

Упростим полученное неравенство. Перенесем из правой части все члены неравенства в левую часть с противоположными знаками.


mx2 − m − 3 + 2x2 mx1
+ m
+ 3
2x1
< 0

Упростим полученное выражение. Некоторые члены неравенства взаимоуничтожатся.


mx2 − mx1
− m + m − 3 + 3 + 2x2 − 2x1

< 0

mx2 − mx1 + 2x2 − 2x1

< 0

Вынесем общие множители за скобки.

m( x2 − x1) + 2(x2 − x1)

< 0

Теперь
вынесем общий множитель

« ( x2 − x1 ) ».

( x2 − x1) (m + 2)

< 0

Вспомним обязательное условие убывания функции.

x2 > x1
y( x2 ) < y( x1 )

Обязательное условие убывания функции

Преобразуем исходное условие убывания функции « x2 > x1 ».
Перенесем все в левую часть.

x2 > x1

x2 − x1 > 0

По условию убывания функции
« x2 − x1 > 0 »,
значит, чтобы
произведение
«( x2 − x1) (m + 2)

» было меньше нуля, требуется, чтобы множитель «(m + 2)» был меньше нуля. Так как по
правилу знаков:
плюс на минус даёт минус.

+ · < 0
(x2 − x1) · (m + 2) < 0

Решим полученное неравенство.

m + 2 < 0
m < −2

Ответ: при «m < −2» функция
« y = mx − m − 3 + 2x »
является убывающей.


Ваши комментарии

Важно!
Галка

Чтобы оставить комментарий, вам нужно войти на наш сайт при помощи

«ВКонтакте».

Пришелец пожимает плечами

Оставить комментарий:


Найти интегралы возрастания и убывания функции. Возрастание и убывание функций, экстремумы

Выпускная работа в форме ЕГЭ для 11-классников обязательно содержит задания на вычисление пределов, промежутков убывания и возрастания производной функции, поиск точек экстремума и построение графиков. Хорошее знание этой темы позволяет правильно ответить на несколько вопросов экзамена и не испытывать затруднений в дальнейшем профессиональном обучении.

Основы дифференциального исчисления – одна из главных тем математики современной школы. Она изучает применение производной для исследования зависимостей переменных – именно через производную можно проанализировать возрастание и убывание функции без обращения к чертежу.

Комплексная подготовка выпускников к сдаче ЕГЭ на образовательном портале «Школково» поможет глубоко понять принципы дифференцирования – подробно разобраться в теории, изучить примеры решения типовых задач и попробовать свои силы в самостоятельной работе. Мы поможем вам ликвидировать пробелы в знаниях – уточнить представление о лексических понятиях темы и зависимостях величин.

Ученики смогут повторить, как находить промежутки монотонности, что значит подъем или убывание производной функции на определенном отрезке, когда граничные точки включаются и не включаются в найденные интервалы.

Прежде чем начинать непосредственное решение тематических задач, мы рекомендуем сначала перейти к разделу «Теоретическая справка» и повторить определения понятий, правила и табличные формулы. Здесь же можно прочитать, как находить и записывать каждый промежуток возрастания и убывания функции на графике производной.

Все предлагаемые сведения излагаются в максимально доступной форме для понимания практически «с нуля». На сайте доступны материалы для восприятия и усвоения в нескольких различных формах – чтения, видеопросмотра и непосредственного тренинга под руководством опытных учителей. Профессиональные педагоги подробно расскажут, как найти промежутки возрастания и убывания производной функции аналитическими и графическими способами. В ходе вебинаров можно будет задать любой интересующий вопрос как по теории, так и по решению конкретных задач.

Вспомнив основные моменты темы, просмотрите примеры на возрастание производной функции, аналогичные заданиям экзаменационных вариантов. Для закрепления усвоенного загляните в «Каталог» — здесь вы найдете практические упражнения для самостоятельной работы. Задания в разделе подобраны разного уровня сложности с учетом наработки навыков. К каждому из них, например, на прилагаются алгоритмы решений и правильные ответы.

Выбирая раздел «Конструктор», учащиеся смогут попрактиковаться в исследовании возрастания и убывания производной функции на реальных вариантах ЕГЭ, постоянно обновляемых с учетом последних изменений и нововведений.

Экстремумы функции

Определение 2

Точка $x_0$ называется точкой максимума функции $f(x)$, если существует такая окрестность данной точки, что для всех $x$ из этой окрестность выполняется неравенство $f(x)le f(x_0)$.

Определение 3

Точка $x_0$ называется точкой максимума функции $f(x)$, если существует такая окрестность данной точки, что для всех $x$ из этой окрестность выполняется неравенство $f(x)ge f(x_0)$.

Понятие экстремума функции тесно связано с понятием критической точки функции. Введем её определение.

Определение 4

$x_0$ называется критической точкой функции $f(x)$, если:

1) $x_0$ — внутренняя точка области определения;

2) $f»left(x_0right)=0$ или не существует.

Для понятия экстремума можно сформулировать теоремы о достаточных и необходимых условиях его существования.

Теорема 2

Достаточное условие экстремума

Пусть точка $x_0$ является критической для функции $y=f(x)$ и лежит в интервале $(a,b)$. Пусть на каждом интервале $left(a,x_0right) и (x_0,b)$ производная $f»(x)$ существует и сохраняет постоянный знак. Тогда:

1) Если на интервале $(a,x_0)$ производная $f»left(xright)>0$, а на интервале $(x_0,b)$ производная $f»left(xright)

2) Если на интервале $(a,x_0)$ производная $f»left(xright)0$, то точка $x_0$ — точка минимума для данной функции.

3) Если и на интервале $(a,x_0)$, и на интервале $(x_0,b)$ производная $f»left(xright) >0$ или производная $f»left(xright)

Данная теорема проиллюстрирована на рисунке 1.

Рисунок 1. Достаточное условие существования экстремумов

Примеры экстремумов (Рис. 2).

Рисунок 2. Примеры точек экстремумов

Правило исследования функции на экстремум

2) Найти производную $f»(x)$;

7) Сделать выводы о наличии максимумов и минимумов на каждом промежутке, используя теорему 2.

Возрастание и убывание функции

Введем, для начала, определения возрастающей и убывающей функций.

Определение 5

Функция $y=f(x)$, определенная на промежутке $X$, называется возрастающей, если для любых точек $x_1,x_2in X$ при $x_1

Определение 6

Функция $y=f(x)$, определенная на промежутке $X$, называется убывающей, если для любых точек $x_1,x_2in X$ при $x_1f(x_2)$.

Исследование функции на возрастание и убывание

Исследовать функции на возрастание и убывание можно с помощью производной.

Для того чтобы исследовать функцию на промежутки возрастания и убывания, необходимо сделать следующее:

1) Найти область определения функции $f(x)$;

2) Найти производную $f»(x)$;

3) Найти точки, в которых выполняется равенство $f»left(xright)=0$;

4) Найти точки, в которых $f»(x)$ не существует;

5) Отметить на координатной прямой все найденные точки и область определения данной функции;

6) Определить знак производной $f»(x)$ на каждом получившемся промежутке;

7) Сделать вывод: на промежутках, где $f»left(xright)0$ функция возрастает. 2-30x+36$;

3) $f»left(xright)=0$;

4) $f»(x)$ существует во всех точках области определения;

5) Координатная прямая:

Рисунок 3.

6) Определить знак производной $f»(x)$ на каждом промежутке:

, если для любой пары точек х
и х»
, а ≤ х выполняется неравенство f
(x
)
f
(
), и строго возрастающей — если выполняется неравенство f
(x
) f
(
). Аналогично определяется убывание и строгое убывание функции. Например, функция у
= х
2 (рис.

, а) строго возрастает на отрезке , а

(рис.

, б) строго убывает на этом отрезке. Возрастающие функции обозначаются f
(x

), а убывающие f
(x
)↓.
Для того чтобы дифференцируемая функция f
(x
) была возрастающей на отрезке [а
, b
], необходимо и достаточно, чтобы её производная f
«(x
) была неотрицательной на [а
, b
].

Наряду с возрастанием и убыванием функции на отрезке рассматривают возрастание и убывание функции в точке. Функция у
= f
(x
) называется возрастающей в точке x
0 , если найдётся такой интервал (α, β), содержащий точку x
0 , что для любой точки х
из (α, β), х>
x
0 , выполняется неравенство f
(x
0)
f
(x
), и для любой точки х
из (α, β), х 0 , выполняется неравенство f
(x
) ≤ f
(x
0). Аналогично определяется строгое возрастание функции в точке x
0 . Если f
«(x


0) >
0, то функция f
(x
) строго возрастает в точке x
0 . Если f
(x
) возрастает в каждой точке интервала (a
, b
), то она возрастает на этом интервале.

С. Б. Стечкин.

Большая советская энциклопедия. — М.: Советская энциклопедия
.
1969-1978
.

Смотреть что такое «Возрастание и убывание функции» в других словарях:

    Понятия математического анализа. Функция f(x) называется возрастающей на отрезке ВОЗРАСТНАЯ СТРУКТУРА НАСЕЛЕНИЯ соотношение численности разных возрастных групп населения. Зависит от уровней рождаемости и смертности, продолжительности жизни людей … Большой Энциклопедический словарь

    Понятия математического анализа. Функция f(х) называется возрастающей на отрезке , если для любой пары точек x1 и x2, a≤x1 … Энциклопедический словарь

    Понятия матем. анализа. Ф ция f(x) наз. возрастающей на отрезке [а, b], если для любой пары точек х1 и x2, а

    Раздел математики, в котором изучаются производные и дифференциалы функций и их применения к исследованию функций. Оформление Д. и. в самостоятельную математическую дисциплину связано с именами И. Ньютона и Г. Лейбница (вторая половина 17 … Большая советская энциклопедия

    Раздел математики, в к ром изучаются понятия производной и дифференциала и способы их применения к исследованию функций. Развитие Д. и. тесно связано с развитием интегрального исчисления. Неразрывно и их содержание. Вместе они составляют основу… … Математическая энциклопедия

    У этого термина существуют и другие значения, см. функция. Запрос «Отображение» перенаправляется сюда; см. также другие значения … Википедия

    Аристотель и перипатетики
    — Аристотелевский вопрос Жизнь Аристотеля Аристотель родился в 384/383 гг. до н. э. в Стагире, на границе с Македонией. Его отец по имени Никомах был врачом на службе у македонского царя Аминта, отца Филиппа. Вместе с семьей молодой Аристотель… … Западная философия от истоков до наших дней

    — (КХД), квантовополевая теория сильного вз ствия кварков и глюонов, построенная по образу квант. электродинамики (КЭД) на основе «цветовой» калибровочной симметрии. В отличие от КЭД, фермионы в КХД имеют дополнит. степень свободы квант. число,… … Физическая энциклопедия

    I Сердце Сердце (лат. соr, греч. cardia) полый фиброзно мышечный орган, который, функционируя как насос, обеспечивает движение крови а системе кровообращения. Анатомия Сердце находится в переднем средостении (Средостение) в Перикарде между… … Медицинская энциклопедия

    Жизнь растения, как и всякого другого живого организма, представляет сложную совокупность взаимосвязанных процессов; наиболее существенный из них, как известно, обмен веществ с окружающей средой.

    Среда является тем источником, откуда… … Биологическая энциклопедия

Монотонность

Очень важным свойством функции является ее монотонность. Зная это свойство различных специальных функций, можно определить поведение различных физических, экономических, социальных и многих других процессов.

Выделяют следующие виды монотонности функций:

1) функция
возрастает
, если на некотором интервале, если для любых двух точек и этого интервала таких, что выполнено, что . Т.е. большему значению аргумента соответствует большее значение функции;

2) функция
убывает
, если на некотором интервале, если для любых двух точек и этого интервала таких, что выполнено, что . Т.е. большему значению аргумента соответствует меньшее значение функции;

3) функция
неубывает
, если на некотором интервале, если для любых двух точек и этого интервала таких, что выполнено, что ;

4) функция
невозрастает
, если на некотором интервале, если для любых двух точек и этого интервала таких, что выполнено, что .

2. Для первых двух случаев еще применяют термин «строгая монотонность».

3. Два последних случая являются специфическими и задаются обычно в виде композиции из нескольких функций.

4. Отдельно отметим, что рассматривать возрастание и убывание графика функции следует именно слева-направо и никак иначе.

2. Четность/нечетность.

Функция называется нечетной
, если при изменении знака аргумента, она меняет свое значение на противоположное. Формульная запись этого выглядит так . Это значит, что после подстановки в функцию на место всех иксов значений «минус икс», функция изменит свой знак. График такой функции симметричен относительно начала координат.

Примерами нечетных функций являются и др.

Например, график действительно обладает симметричностью относительно начала координат:

Функция называется четной
, если при изменении знака аргумента, она не меняет свое значение. Формульная запись этого выглядит так . Это значит, что после подстановки в функцию на место всех иксов значений «минус икс», функция в результате не изменится. График такой функции симметричен относительно оси .

Примерами четных функций являются и др.

К примеру, покажем симметричность графика относительно оси :

Если функция не относится ни к одному из указанных видов, то ее называют ни четной ни нечетной или функцией общего вида
. У таких функций нет симметрии.

Такой функцией, например, является недавно рассмотренная нами линейная функция с графиком:

3. Особым свойством функций является периодичность.

Дело в том, что периодичными функциями, которые рассматриваются в стандартной школьной программе, являются только тригонометрические функции. Мы уже подробно о них говорили при изучении соответствующей темы.

Периодичная функция
– это функция, которая не меняет свои значения при добавлении к аргументу определенного постоянного ненулевого числа.

Такое минимальное число называют периодом функции
и обозначают буквой .

Формульная запись этого выглядит следующим образом: .

Посмотрим на это свойство на примере графика синуса:

Вспомним, что периодом функций и является , а периодом и – .

Как мы уже знаем, для тригонометрических функций со сложным аргументом может быть нестандартный период. Речь идет о функциях вида:

У них период равен . И о функциях:

У них период равен .

Как видим, для вычисления нового периода стандартный период просто делится на множитель при аргументе. От остальных видоизменений функции он не зависит.

Ограниченность.

Функцию
y=f(x)называют ограниченной снизу на множестве Х⊂D(f), если существует такое число а, что для любых хϵХ выполняется неравенство f(x)

Функцию
y=f(x)называют ограниченной сверху на множестве Х⊂D(f), если существует такое число а, что для любых хϵХ выполняется неравенство f(x)

Если промежуток Х не указывается, то считают, что функция ограничена на всей области определения. Функция ограниченная и сверху, и снизу называется ограниченной.

Ограниченность функции легко читается по графику. Можно провести некоторую прямую у=а, и если функция выше этой прямой, то ограниченность снизу.

Если ниже, то соответственно сверху. Ниже представлен график ограниченной снизу функции. График ограниченной функции, ребята, попробуйте нарисовать сами.

Тема: Свойства функций: промежутки возрастания и убывания; наибольшее и наименьшее значения; точки экстремума (локального максимума и минимума), выпуклость функции.

Промежутки возрастания и убывания.

На основании достаточных условий (признаков) возрастания и убывания функции находятся промежутки возрастания и убывания функции.

Вот формулировки признаков возрастания и убывания функции на интервале:

· если производная функции y=f(x)
положительна для любого x
из интервала X
, то функция возрастает на X
;

· если производная функции y=f(x)
отрицательна для любого x
из интервала X
, то функция убывает на X
.

Таким образом, чтобы определить промежутки возрастания и убывания функции необходимо:

· найти область определения функции;

· найти производную функции;

· решить неравенства и на области определения;

Как определить функция возрастает или убывает примеры. Промежутки возрастания и убывания

Монотонность

Очень важным свойством функции является ее монотонность. Зная это свойство различных специальных функций, можно определить поведение различных физических, экономических, социальных и многих других процессов.

Выделяют следующие виды монотонности функций:

1) функция
возрастает
, если на некотором интервале, если для любых двух точек и этого интервала таких, что выполнено, что . Т.е. большему значению аргумента соответствует большее значение функции;

2) функция
убывает
, если на некотором интервале, если для любых двух точек и этого интервала таких, что выполнено, что . Т. е. большему значению аргумента соответствует меньшее значение функции;

3) функция
неубывает
, если на некотором интервале, если для любых двух точек и этого интервала таких, что выполнено, что ;

4) функция
невозрастает
, если на некотором интервале, если для любых двух точек и этого интервала таких, что выполнено, что .

2. Для первых двух случаев еще применяют термин «строгая монотонность».

3. Два последних случая являются специфическими и задаются обычно в виде композиции из нескольких функций.

4. Отдельно отметим, что рассматривать возрастание и убывание графика функции следует именно слева-направо и никак иначе.

2. Четность/нечетность.

Функция называется нечетной
, если при изменении знака аргумента, она меняет свое значение на противоположное. Формульная запись этого выглядит так . Это значит, что после подстановки в функцию на место всех иксов значений «минус икс», функция изменит свой знак. График такой функции симметричен относительно начала координат.

Примерами нечетных функций являются и др.

Например, график действительно обладает симметричностью относительно начала координат:

Функция называется четной
, если при изменении знака аргумента, она не меняет свое значение. Формульная запись этого выглядит так . Это значит, что после подстановки в функцию на место всех иксов значений «минус икс», функция в результате не изменится. График такой функции симметричен относительно оси .

Примерами четных функций являются и др.

К примеру, покажем симметричность графика относительно оси :

Если функция не относится ни к одному из указанных видов, то ее называют ни четной ни нечетной или функцией общего вида
. У таких функций нет симметрии.

Такой функцией, например, является недавно рассмотренная нами линейная функция с графиком:

3. Особым свойством функций является периодичность.

Дело в том, что периодичными функциями, которые рассматриваются в стандартной школьной программе, являются только тригонометрические функции. Мы уже подробно о них говорили при изучении соответствующей темы.

Периодичная функция
– это функция, которая не меняет свои значения при добавлении к аргументу определенного постоянного ненулевого числа.

Такое минимальное число называют периодом функции
и обозначают буквой .

Формульная запись этого выглядит следующим образом: .

Посмотрим на это свойство на примере графика синуса:

Вспомним, что периодом функций и является , а периодом и – .

Как мы уже знаем, для тригонометрических функций со сложным аргументом может быть нестандартный период. Речь идет о функциях вида:

У них период равен . И о функциях:

У них период равен .

Как видим, для вычисления нового периода стандартный период просто делится на множитель при аргументе. От остальных видоизменений функции он не зависит.

Ограниченность.

Функцию
y=f(x)называют ограниченной снизу на множестве Х⊂D(f), если существует такое число а, что для любых хϵХ выполняется неравенство f(x)

Функцию
y=f(x)называют ограниченной сверху на множестве Х⊂D(f), если существует такое число а, что для любых хϵХ выполняется неравенство f(x)

Если промежуток Х не указывается, то считают, что функция ограничена на всей области определения. Функция ограниченная и сверху, и снизу называется ограниченной.

Ограниченность функции легко читается по графику. Можно провести некоторую прямую у=а, и если функция выше этой прямой, то ограниченность снизу.

Если ниже, то соответственно сверху. Ниже представлен график ограниченной снизу функции. График ограниченной функции, ребята, попробуйте нарисовать сами.

Тема: Свойства функций: промежутки возрастания и убывания; наибольшее и наименьшее значения; точки экстремума (локального максимума и минимума), выпуклость функции.

Промежутки возрастания и убывания.

На основании достаточных условий (признаков) возрастания и убывания функции находятся промежутки возрастания и убывания функции.

Вот формулировки признаков возрастания и убывания функции на интервале:

· если производная функции y=f(x)
положительна для любого x
из интервала X
, то функция возрастает на X
;

· если производная функции y=f(x)
отрицательна для любого x
из интервала X
, то функция убывает на X
.

Таким образом, чтобы определить промежутки возрастания и убывания функции необходимо:

· найти область определения функции;

· найти производную функции;

· решить неравенства и на области определения;

Пусть на некоторой плоскости задана прямоугольная система координат. Графиком некоторой функции , (X- область определения) называется множество точек этой плоскости с координатами, где .

Для построения графика нужно изобразить на плоскости множество точек, координаты которых (x;y) связаны соотношением .

Чаще всего графиком функции является некоторая кривая.

Самый простой способ построения графика — построение по точкам.

Составляется таблица, в которой в одной ячейке стоит значение аргумента, а в противоположной ей значение функции от этого аргумента. Затем полученные точки отмечаются на плоскости, и через них проводится кривая.

Пример построения по точкам графика функции :

Построим таблицу.

Теперь строим график.

Но таким способом не всегда возможно построить достаточно точный график — для точности нужно брать очень много точек. Поэтому используют различные методы исследования функции.

С полной схемой исследования функции знакомятся в высших учебных заведениях. Одним из пунктов исследования функции является нахождение промежутков возрастания (убывания) функции.

Функция называется возрастающей (убывающей) на некотором промежутке, если , для любых x 2 и x 1 из этого промежутка, таких, что x 2 >x 1 .

Например, функция, график которой изображен на следующем рисунке, на промежутках возрастает, а на промежутке (-5;3) убывает. То есть, на промежутках график идет «в гору». А на промежутке (-5;3) «под гору».

Еще одним из пунктов исследования функции является исследование функции на периодичность.

Функция называется периодичной, если существует такое число T, что .

Число T называют периодом функции. Например, функция периодична, здесь период равен 2П, так

Примеры графиков периодичных функций:

Период первой функции равен 3, а второй – 4.

Функция называется четной, если Пример четной функции y=x 2 .

Функция называется нечетной, если Пример нечетной функции y=x 3 .

График четной функции симметричен относительно оси ОУ (осевая симметрия).

График нечетной функции симметричен относительно начала координат (центральная симметрия).

Примеры графиков четной (слева) и нечетной (справа) функции.

Чтобы определить характер функции и говорить о ее поведении, необходимо находить промежутки возрастания и убывания. Этот процесс получил название исследования функции и построения графика. Точка экстремума используется при нахождении наибольшего и наименьшего значения функции, так как в них происходит возрастание или убывание функции из интервала.

Данная статья раскрывает определения, формулируем достаточный признак возрастания и убывания на интервале и условие существования экстремума. Это применимо к решению примеров и задач. Следует повторить раздел дифференцирования функций, потому как при решении необходимо будет использовать нахождение производной.

Yandex.RTB R-A-339285-1
Определение 1

Функция y = f (x) будет возрастать на интервале x , когда при любых x 1 ∈ X и x 2 ∈ X , x 2 > x 1 неравенство f (x 2) > f (x 1) будет выполнимо. Иначе говоря, большему значению аргумента соответствует большее значение функции.

Определение 2

Функция y = f (x) считается убывающей на интервале x , когда при любых x 1 ∈ X , x 2 ∈ X , x 2 > x 1 равенство f (x 2) > f (x 1) считается выполнимым. Иначе говоря, большему значению функции соответствует меньшее значение аргумента. Рассмотрим рисунок, приведенный ниже.

Замечание:

Когда функция определенная и непрерывная в концах интервала возрастания и убывания, то есть (a ; b) , где х = а, х = b , точки включены в промежуток возрастания и убывания. Определению это не противоречит, значит, имеет место быть на промежутке x .

Основные свойства элементарных функций типа y = sin x – определенность и непрерывность при действительных значениях аргументах. Отсюда получаем, что возрастание синуса происходит на интервале — π 2 ; π 2 , тогда возрастание на отрезке имеет вид — π 2 ; π 2 .

Определение 3

Точка х 0 называется точкой максимума
для функции y = f (x) , когда для всех значений x неравенство f (x 0) ≥ f (x) является справедливым. Максимум функции
– это значение функции в точке, причем обозначается y m a x .

Точка х 0 называется точкой минимума для функции y = f (x) , когда для всех значений x неравенство f (x 0) ≤ f (x) является справедливым. Минимум функции
– это значение функции в точке, причем имеет обозначение вида y m i n .

Окрестностями точки х 0 считаются точки экстремума,
а значение функции, которое соответствует точкам экстремума. Рассмотрим рисунок, приведенный ниже.

Экстремумы функции с набольшим и с наименьшим значением функции. Рассмотрим рисунок, приведенный ниже.

Первый рисунок говорит о том, что необходимо найти наибольшее значение функции из отрезка [ a ; b ] . Оно находится при помощи точек максимума и равняется максимальному значению функции, а второй рисунок больше походит на поиск точки максимума при х = b .

Достаточные условия возрастания и убывания функции

Чтобы найти максимумы и минимумы функции, необходимо применять признаки экстремума в том случае, когда функция удовлетворяет этим условиям. Самым часто используемым считается первый признак.

Первое достаточное условие экстремума

Определение 4

Пусть задана функция y = f (x) , которая дифференцируема в ε окрестности точки x 0 , причем имеет непрерывность в заданной точке x 0 . Отсюда получаем, что

  • когда f » (x) > 0 с x ∈ (x 0 — ε ; x 0) и f » (x)
  • когда f » (x) 0 при x ∈ (x 0 ; x 0 + ε) , тогда x 0 является точкой минимума.

Иначе говоря, получим их условия постановки знака:

  • когда функция непрерывна в точке x 0 , тогда имеет производную с меняющимся знаком, то есть с + на — , значит, точка называется максимумом;
  • когда функция непрерывна в точке x 0 , тогда имеет производную с меняющимся знаком с — на + , значит, точка называется минимумом.

Чтобы верно определить точки максимума и минимума функции, необходимо следовать алгоритму их нахождения:

  • найти область определения;
  • найти производную функции на этой области;
  • определить нули и точки, где функция не существует;
  • определение знака производной на интервалах;
  • выбрать точки, где функция меняет знак.

Рассмотрим алгоритм на примере решения нескольких примеров на нахождение экстремумов функции.

Пример 1

Найти точки максимума и минимума заданной функции y = 2 (x + 1) 2 x — 2 .

Решение

Область определения данной функции – это все действительные числа кроме х = 2 . Для начала найдем производную функции и получим:

y » = 2 x + 1 2 x — 2 » = 2 · x + 1 2 » · (x — 2) — (x + 1) 2 · (x — 2) » (x — 2) 2 = = 2 · 2 · (x + 1) · (x + 1) » · (x — 2) — (x + 1) 2 · 1 (x — 2) 2 = 2 · 2 · (x + 1) · (x — 2) — (x + 2) 2 (x — 2) 2 = = 2 · (x + 1) · (x — 5) (x — 2) 2

Отсюда видим, что нули функции – это х = — 1 , х = 5 , х = 2 , то есть каждую скобку необходимо приравнять к нулю. Отметим на числовой оси и получим:

Теперь определим знаки производной из каждого интервала. Необходимо выбрать точку, входящую в интервал, подставить в выражение. Например, точки х = — 2 , х = 0 , х = 3 , х = 6 .

Получаем, что

y » (- 2) = 2 · (x + 1) · (x — 5) (x — 2) 2 x = — 2 = 2 · (- 2 + 1) · (- 2 — 5) (- 2 — 2) 2 = 2 · 7 16 = 7 8 > 0 , значит, интервал — ∞ ; — 1 имеет положительную производную. Аналогичным образом получаем, что

y » (0) = 2 · (0 + 1) · 0 — 5 0 — 2 2 = 2 · — 5 4 = — 5 2 0

Так как второй интервал получился меньше нуля, значит, производная на отрезке будет отрицательной. Третий с минусом, четвертый с плюсом. Для определения непрерывности необходимо обратить внимание на знак производной, если он меняется, тогда это точка экстремума.

Получим, что в точке х = — 1 функция будет непрерывна, значит, производная изменит знак с + на — . По первому признаку имеем, что х = — 1 является точкой максимума, значит получаем

y m a x = y (- 1) = 2 · (x + 1) 2 x — 2 x = — 1 = 2 · (- 1 + 1) 2 — 1 — 2 = 0

Точка х = 5 указывает на то, что функция является непрерывной, а производная поменяет знак с – на +. Значит, х=-1 является точкой минимума, причем ее нахождение имеет вид

y m i n = y (5) = 2 · (x + 1) 2 x — 2 x = 5 = 2 · (5 + 1) 2 5 — 2 = 24

Графическое изображение

Ответ:
y m a x = y (- 1) = 0 , y m i n = y (5) = 24 .

Стоит обратить внимание на то, что использование первого достаточного признака экстремума не требует дифференцируемости функции с точке x 0 , этим и упрощает вычисление.

Пример 2

Найти точки максимума и минимума функции y = 1 6 x 3 = 2 x 2 + 22 3 x — 8 .

Решение.

Область определения функции – это все действительные числа. Это можно записать в виде системы уравнений вида:

1 6 x 3 — 2 x 2 — 22 3 x — 8 , x

После чего необходимо найти производную:

y » = 1 6 x 3 — 2 x 2 — 22 3 x — 8 » , x 0 y » = — 1 2 x 2 — 4 x — 22 3 , x 0

Точка х = 0 не имеет производной, потому как значения односторонних пределов разные. Получим, что:

lim y » x → 0 — 0 = lim y x → 0 — 0 — 1 2 x 2 — 4 x — 22 3 = — 1 2 · (0 — 0) 2 — 4 · (0 — 0) — 22 3 = — 22 3 lim y » x → 0 + 0 = lim y x → 0 — 0 1 2 x 2 — 4 x + 22 3 = 1 2 · (0 + 0) 2 — 4 · (0 + 0) + 22 3 = + 22 3

Отсюда следует, что функция непрерывна в точке х = 0 , тогда вычисляем

lim y x → 0 — 0 = lim x → 0 — 0 — 1 6 x 3 — 2 x 2 — 22 3 x — 8 = = — 1 6 · (0 — 0) 3 — 2 · (0 — 0) 2 — 22 3 · (0 — 0) — 8 = — 8 lim y x → 0 + 0 = lim x → 0 — 0 1 6 x 3 — 2 x 2 + 22 3 x — 8 = = 1 6 · (0 + 0) 3 — 2 · (0 + 0) 2 + 22 3 · (0 + 0) — 8 = — 8 y (0) = 1 6 x 3 — 2 x 2 + 22 3 x — 8 x = 0 = 1 6 · 0 3 — 2 · 0 2 + 22 3 · 0 — 8 = — 8

Необходимо произвести вычисления для нахождения значения аргумента, когда производная становится равной нулю:

1 2 x 2 — 4 x — 22 3 , x

1 2 x 2 — 4 x + 22 3 , x > 0 D = (- 4) 2 — 4 · 1 2 · 22 3 = 4 3 x 3 = 4 + 4 3 2 · 1 2 = 4 + 2 3 3 > 0 x 4 = 4 — 4 3 2 · 1 2 = 4 — 2 3 3 > 0

Все полученные точки нужно отметить на прямой для определения знака каждого интервала. Поэтому необходимо вычислить производную в произвольных точках у каждого интервала. Например, у нас можно взять точки со значениями x = — 6 , x = — 4 , x = — 1 , x = 1 , x = 4 , x = 6 . Получим, что

y » (- 6) = — 1 2 x 2 — 4 x — 22 3 x = — 6 = — 1 2 · — 6 2 — 4 · (- 6) — 22 3 = — 4 3 0 y » (- 1) = — 1 2 x 2 — 4 x — 22 3 x = — 1 = — 1 2 · (- 1) 2 — 4 · (- 1) — 22 3 = 23 6 0 y » (4) = 1 2 x 2 — 4 x + 22 3 x = 4 = 1 2 · 4 2 — 4 · 4 + 22 3 = — 2 3 0

Изображение на прямой имеет вид

Значит, приходим к тому, что необходимо прибегнуть к первому признаку экстремума. Вычислим и получим, что

x = — 4 — 2 3 3 , x = 0 , x = 4 + 2 3 3 , тогда отсюда точки максимума имеют значени x = — 4 + 2 3 3 , x = 4 — 2 3 3

Перейдем к вычислению минимумов:

y m i n = y — 4 — 2 3 3 = 1 6 x 3 — 2 2 + 22 3 x — 8 x = — 4 — 2 3 3 = — 8 27 3 y m i n = y (0) = 1 6 x 3 — 2 2 + 22 3 x — 8 x = 0 = — 8 y m i n = y 4 + 2 3 3 = 1 6 x 3 — 2 2 + 22 3 x — 8 x = 4 + 2 3 3 = — 8 27 3

Произведем вычисления максимумов функции. Получим, что

y m a x = y — 4 + 2 3 3 = 1 6 x 3 — 2 2 + 22 3 x — 8 x = — 4 + 2 3 3 = 8 27 3 y m a x = y 4 — 2 3 3 = 1 6 x 3 — 2 2 + 22 3 x — 8 x = 4 — 2 3 3 = 8 27 3

Графическое изображение

Ответ:

y m i n = y — 4 — 2 3 3 = — 8 27 3 y m i n = y (0) = — 8 y m i n = y 4 + 2 3 3 = — 8 27 3 y m a x = y — 4 + 2 3 3 = 8 27 3 y m a x = y 4 — 2 3 3 = 8 27 3

Если задана функция f » (x 0) = 0 , тогда при ее f «» (x 0) > 0 получаем, что x 0 является точкой минимума, если f «» (x 0)

Пример 3

Найти максимумы и минимумы функции y = 8 x x + 1 .

Решение

Для начала находим область определения. Получаем, что

D (y) : x ≥ 0 x ≠ — 1 ⇔ x ≥ 0

Необходимо продифференцировать функцию, после чего получим

y » = 8 x x + 1 » = 8 · x » · (x + 1) — x · (x + 1) » (x + 1) 2 = = 8 · 1 2 x · (x + 1) — x · 1 (x + 1) 2 = 4 · x + 1 — 2 x (x + 1) 2 · x = 4 · — x + 1 (x + 1) 2 · x

При х = 1 производная становится равной нулю, значит, точка является возможным экстремумом. Для уточнения необходимо найти вторую производную и вычислить значение при х = 1 . Получаем:

y «» = 4 · — x + 1 (x + 1) 2 · x » = = 4 · (- x + 1) » · (x + 1) 2 · x — (- x + 1) · x + 1 2 · x » (x + 1) 4 · x = = 4 · (- 1) · (x + 1) 2 · x — (- x + 1) · x + 1 2 » · x + (x + 1) 2 · x » (x + 1) 4 · x = = 4 · — (x + 1) 2 x — (- x + 1) · 2 x + 1 (x + 1) » x + (x + 1) 2 2 x (x + 1) 4 · x = = — (x + 1) 2 x — (- x + 1) · x + 1 · 2 x + x + 1 2 x (x + 1) 4 · x = = 2 · 3 x 2 — 6 x — 1 x + 1 3 · x 3 ⇒ y «» (1) = 2 · 3 · 1 2 — 6 · 1 — 1 (1 + 1) 3 · (1) 3 = 2 · — 4 8 = — 1

Значит, использовав 2 достаточное условие экстремума, получаем, что х = 1 является точкой максимума. Иначе запись имеет вид y m a x = y (1) = 8 1 1 + 1 = 4 .

Графическое изображение

Ответ:
y m a x = y (1) = 4 ..

Определение 5

Функция y = f (x) имеет ее производную до n -го порядка в ε окрестности заданной точки x 0 и производную до n + 1 -го порядка в точке x 0 . Тогда f » (x 0) = f «» (x 0) = f » » » (x 0) = . . . = f n (x 0) = 0 .

Отсюда следует, что когда n является четным числом, то x 0 считается точкой перегиба, когда n является нечетным числом, то x 0 точка экстремума, причем f (n + 1) (x 0) > 0 , тогда x 0 является точкой минимума, f (n + 1) (x 0)

Пример 4

Найти точки максимума и минимума функции y y = 1 16 (x + 1) 3 (x — 3) 4 .

Решение

Исходная функция – целая рациональная, отсюда следует, что область определения – все действительные числа. Необходимо продифференцировать функцию. Получим, что

y » = 1 16 x + 1 3 » (x — 3) 4 + (x + 1) 3 x — 3 4 » = = 1 16 (3 (x + 1) 2 (x — 3) 4 + (x + 1) 3 4 (x — 3) 3) = = 1 16 (x + 1) 2 (x — 3) 3 (3 x — 9 + 4 x + 4) = 1 16 (x + 1) 2 (x — 3) 3 (7 x — 5)

Данная производная обратится в ноль при x 1 = — 1 , x 2 = 5 7 , x 3 = 3 . То есть точки могут быть точками возможного экстремума. Необходимо применить третье достаточное условие экстремума. Нахождение второй производной позволяет в точности определить наличие максимума и минимума функции. Вычисление второй производной производится в точках ее возможного экстремума. Получаем, что

y «» = 1 16 x + 1 2 (x — 3) 3 (7 x — 5) » = 1 8 (x + 1) (x — 3) 2 (21 x 2 — 30 x — 3) y «» (- 1) = 0 y «» 5 7 = — 36864 2401

Значит, что x 2 = 5 7 является точкой максимума. Применив 3 достаточный признак, получаем, что при n = 1 и f (n + 1) 5 7

Необходимо определить характер точек x 1 = — 1 , x 3 = 3 . Для этого необходимо найти третью производную, вычислить значения в этих точках. Получаем, что

y » » » = 1 8 (x + 1) (x — 3) 2 (21 x 2 — 30 x — 3) » = = 1 8 (x — 3) (105 x 3 — 225 x 2 — 45 x + 93) y » » » (- 1) = 96 ≠ 0 y » » » (3) = 0

Значит, x 1 = — 1 является точкой перегиба функции, так как при n = 2 и f (n + 1) (- 1) ≠ 0 . Необходимо исследовать точку x 3 = 3 . Для этого находим 4 производную и производим вычисления в этой точке:

y (4) = 1 8 (x — 3) (105 x 3 — 225 x 2 — 45 x + 93) » = = 1 2 (105 x 3 — 405 x 2 + 315 x + 57) y (4) (3) = 96 > 0

Из выше решенного делаем вывод, что x 3 = 3 является точкой минимума функции.

Графическое изображение

Ответ:
x 2 = 5 7 является точкой максимума, x 3 = 3 — точкой минимума заданной функции.

Если вы заметили ошибку в тексте, пожалуйста, выделите её и нажмите Ctrl+Enter

Выпускная работа в форме ЕГЭ для 11-классников обязательно содержит задания на вычисление пределов, промежутков убывания и возрастания производной функции, поиск точек экстремума и построение графиков. Хорошее знание этой темы позволяет правильно ответить на несколько вопросов экзамена и не испытывать затруднений в дальнейшем профессиональном обучении.

Основы дифференциального исчисления – одна из главных тем математики современной школы. Она изучает применение производной для исследования зависимостей переменных – именно через производную можно проанализировать возрастание и убывание функции без обращения к чертежу.

Комплексная подготовка выпускников к сдаче ЕГЭ на образовательном портале «Школково» поможет глубоко понять принципы дифференцирования – подробно разобраться в теории, изучить примеры решения типовых задач и попробовать свои силы в самостоятельной работе. Мы поможем вам ликвидировать пробелы в знаниях – уточнить представление о лексических понятиях темы и зависимостях величин. Ученики смогут повторить, как находить промежутки монотонности, что значит подъем или убывание производной функции на определенном отрезке, когда граничные точки включаются и не включаются в найденные интервалы.

Прежде чем начинать непосредственное решение тематических задач, мы рекомендуем сначала перейти к разделу «Теоретическая справка» и повторить определения понятий, правила и табличные формулы. Здесь же можно прочитать, как находить и записывать каждый промежуток возрастания и убывания функции на графике производной.

Все предлагаемые сведения излагаются в максимально доступной форме для понимания практически «с нуля». На сайте доступны материалы для восприятия и усвоения в нескольких различных формах – чтения, видеопросмотра и непосредственного тренинга под руководством опытных учителей. Профессиональные педагоги подробно расскажут, как найти промежутки возрастания и убывания производной функции аналитическими и графическими способами. В ходе вебинаров можно будет задать любой интересующий вопрос как по теории, так и по решению конкретных задач.

Вспомнив основные моменты темы, просмотрите примеры на возрастание производной функции, аналогичные заданиям экзаменационных вариантов. Для закрепления усвоенного загляните в «Каталог» — здесь вы найдете практические упражнения для самостоятельной работы. Задания в разделе подобраны разного уровня сложности с учетом наработки навыков. К каждому из них, например, на прилагаются алгоритмы решений и правильные ответы.

Выбирая раздел «Конструктор», учащиеся смогут попрактиковаться в исследовании возрастания и убывания производной функции на реальных вариантах ЕГЭ, постоянно обновляемых с учетом последних изменений и нововведений.

Экстремумы функции

Определение 2

Точка $x_0$ называется точкой максимума функции $f(x)$, если существует такая окрестность данной точки, что для всех $x$ из этой окрестность выполняется неравенство $f(x)le f(x_0)$.

Определение 3

Точка $x_0$ называется точкой максимума функции $f(x)$, если существует такая окрестность данной точки, что для всех $x$ из этой окрестность выполняется неравенство $f(x)ge f(x_0)$.

Понятие экстремума функции тесно связано с понятием критической точки функции. Введем её определение.

Определение 4

$x_0$ называется критической точкой функции $f(x)$, если:

1) $x_0$ — внутренняя точка области определения;

2) $f»left(x_0right)=0$ или не существует.

Для понятия экстремума можно сформулировать теоремы о достаточных и необходимых условиях его существования.

Теорема 2

Достаточное условие экстремума

Пусть точка $x_0$ является критической для функции $y=f(x)$ и лежит в интервале $(a,b)$. Пусть на каждом интервале $left(a,x_0right) и (x_0,b)$ производная $f»(x)$ существует и сохраняет постоянный знак. Тогда:

1) Если на интервале $(a,x_0)$ производная $f»left(xright)>0$, а на интервале $(x_0,b)$ производная $f»left(xright)

2) Если на интервале $(a,x_0)$ производная $f»left(xright)0$, то точка $x_0$ — точка минимума для данной функции.

3) Если и на интервале $(a,x_0)$, и на интервале $(x_0,b)$ производная $f»left(xright) >0$ или производная $f»left(xright)

Данная теорема проиллюстрирована на рисунке 1.

Рисунок 1. Достаточное условие существования экстремумов

Примеры экстремумов (Рис. 2).

Рисунок 2. Примеры точек экстремумов

Правило исследования функции на экстремум

2) Найти производную $f»(x)$;

7) Сделать выводы о наличии максимумов и минимумов на каждом промежутке, используя теорему 2.

Возрастание и убывание функции

Введем, для начала, определения возрастающей и убывающей функций.

Определение 5

Функция $y=f(x)$, определенная на промежутке $X$, называется возрастающей, если для любых точек $x_1,x_2in X$ при $x_1

Определение 6

Функция $y=f(x)$, определенная на промежутке $X$, называется убывающей, если для любых точек $x_1,x_2in X$ при $x_1f(x_2)$.

Исследование функции на возрастание и убывание

Исследовать функции на возрастание и убывание можно с помощью производной.

Для того чтобы исследовать функцию на промежутки возрастания и убывания, необходимо сделать следующее:

1) Найти область определения функции $f(x)$;

2) Найти производную $f»(x)$;

3) Найти точки, в которых выполняется равенство $f»left(xright)=0$;

4) Найти точки, в которых $f»(x)$ не существует;

5) Отметить на координатной прямой все найденные точки и область определения данной функции;

6) Определить знак производной $f»(x)$ на каждом получившемся промежутке;

7) Сделать вывод: на промежутках, где $f»left(xright)0$ функция возрастает. 2-30x+36$;

3) $f»left(xright)=0$;

4) $f»(x)$ существует во всех точках области определения;

5) Координатная прямая:

Рисунок 3.

6) Определить знак производной $f»(x)$ на каждом промежутке:

}

При каком условии функция возрастает. Возрастание и убывание функции на интервале, экстремумы. необходимым условием экстремума

Монотонность

Очень важным свойством функции является ее монотонность. Зная это свойство различных специальных функций, можно определить поведение различных физических, экономических, социальных и многих других процессов.

Выделяют следующие виды монотонности функций:

1) функция
возрастает
, если на некотором интервале, если для любых двух точек и этого интервала таких, что выполнено, что . Т.е. большему значению аргумента соответствует большее значение функции;

2) функция
убывает
, если на некотором интервале, если для любых двух точек и этого интервала таких, что выполнено, что . Т.е. большему значению аргумента соответствует меньшее значение функции;

3) функция
неубывает
, если на некотором интервале, если для любых двух точек и этого интервала таких, что выполнено, что ;

4) функция
невозрастает
, если на некотором интервале, если для любых двух точек и этого интервала таких, что выполнено, что .

2. Для первых двух случаев еще применяют термин «строгая монотонность».

3. Два последних случая являются специфическими и задаются обычно в виде композиции из нескольких функций.

4. Отдельно отметим, что рассматривать возрастание и убывание графика функции следует именно слева-направо и никак иначе.

2. Четность/нечетность.

Функция называется нечетной
, если при изменении знака аргумента, она меняет свое значение на противоположное. Формульная запись этого выглядит так . Это значит, что после подстановки в функцию на место всех иксов значений «минус икс», функция изменит свой знак. График такой функции симметричен относительно начала координат.

Примерами нечетных функций являются и др.

Например, график действительно обладает симметричностью относительно начала координат:

Функция называется четной
, если при изменении знака аргумента, она не меняет свое значение. Формульная запись этого выглядит так . Это значит, что после подстановки в функцию на место всех иксов значений «минус икс», функция в результате не изменится. График такой функции симметричен относительно оси .

Примерами четных функций являются и др.

К примеру, покажем симметричность графика относительно оси :

Если функция не относится ни к одному из указанных видов, то ее называют ни четной ни нечетной или функцией общего вида
. У таких функций нет симметрии.

Такой функцией, например, является недавно рассмотренная нами линейная функция с графиком:

3. Особым свойством функций является периодичность.

Дело в том, что периодичными функциями, которые рассматриваются в стандартной школьной программе, являются только тригонометрические функции. Мы уже подробно о них говорили при изучении соответствующей темы.

Периодичная функция
– это функция, которая не меняет свои значения при добавлении к аргументу определенного постоянного ненулевого числа.

Такое минимальное число называют периодом функции
и обозначают буквой .

Формульная запись этого выглядит следующим образом: .

Посмотрим на это свойство на примере графика синуса:

Вспомним, что периодом функций и является , а периодом и – .

Как мы уже знаем, для тригонометрических функций со сложным аргументом может быть нестандартный период. Речь идет о функциях вида:

У них период равен . И о функциях:

У них период равен .

Как видим, для вычисления нового периода стандартный период просто делится на множитель при аргументе. От остальных видоизменений функции он не зависит.

Ограниченность.

Функцию
y=f(x)называют ограниченной снизу на множестве Х⊂D(f), если существует такое число а, что для любых хϵХ выполняется неравенство f(x)

Функцию
y=f(x)называют ограниченной сверху на множестве Х⊂D(f), если существует такое число а, что для любых хϵХ выполняется неравенство f(x)

Если промежуток Х не указывается, то считают, что функция ограничена на всей области определения. Функция ограниченная и сверху, и снизу называется ограниченной.

Ограниченность функции легко читается по графику. Можно провести некоторую прямую у=а, и если функция выше этой прямой, то ограниченность снизу.

Если ниже, то соответственно сверху. Ниже представлен график ограниченной снизу функции. График ограниченной функции, ребята, попробуйте нарисовать сами.

Тема: Свойства функций: промежутки возрастания и убывания; наибольшее и наименьшее значения; точки экстремума (локального максимума и минимума), выпуклость функции.

Промежутки возрастания и убывания.

На основании достаточных условий (признаков) возрастания и убывания функции находятся промежутки возрастания и убывания функции.

Вот формулировки признаков возрастания и убывания функции на интервале:

· если производная функции y=f(x)
положительна для любого x
из интервала X
, то функция возрастает на X
;

· если производная функции y=f(x)
отрицательна для любого x
из интервала X
, то функция убывает на X
.

Таким образом, чтобы определить промежутки возрастания и убывания функции необходимо:

· найти область определения функции;

· найти производную функции;

· решить неравенства и на области определения;

Выпускная работа в форме ЕГЭ для 11-классников обязательно содержит задания на вычисление пределов, промежутков убывания и возрастания производной функции, поиск точек экстремума и построение графиков. Хорошее знание этой темы позволяет правильно ответить на несколько вопросов экзамена и не испытывать затруднений в дальнейшем профессиональном обучении.

Основы дифференциального исчисления – одна из главных тем математики современной школы. Она изучает применение производной для исследования зависимостей переменных – именно через производную можно проанализировать возрастание и убывание функции без обращения к чертежу.

Комплексная подготовка выпускников к сдаче ЕГЭ на образовательном портале «Школково» поможет глубоко понять принципы дифференцирования – подробно разобраться в теории, изучить примеры решения типовых задач и попробовать свои силы в самостоятельной работе. Мы поможем вам ликвидировать пробелы в знаниях – уточнить представление о лексических понятиях темы и зависимостях величин. Ученики смогут повторить, как находить промежутки монотонности, что значит подъем или убывание производной функции на определенном отрезке, когда граничные точки включаются и не включаются в найденные интервалы.

Прежде чем начинать непосредственное решение тематических задач, мы рекомендуем сначала перейти к разделу «Теоретическая справка» и повторить определения понятий, правила и табличные формулы. Здесь же можно прочитать, как находить и записывать каждый промежуток возрастания и убывания функции на графике производной.

Все предлагаемые сведения излагаются в максимально доступной форме для понимания практически «с нуля». На сайте доступны материалы для восприятия и усвоения в нескольких различных формах – чтения, видеопросмотра и непосредственного тренинга под руководством опытных учителей. Профессиональные педагоги подробно расскажут, как найти промежутки возрастания и убывания производной функции аналитическими и графическими способами. В ходе вебинаров можно будет задать любой интересующий вопрос как по теории, так и по решению конкретных задач.

Вспомнив основные моменты темы, просмотрите примеры на возрастание производной функции, аналогичные заданиям экзаменационных вариантов. Для закрепления усвоенного загляните в «Каталог» — здесь вы найдете практические упражнения для самостоятельной работы. Задания в разделе подобраны разного уровня сложности с учетом наработки навыков. К каждому из них, например, на прилагаются алгоритмы решений и правильные ответы.

Выбирая раздел «Конструктор», учащиеся смогут попрактиковаться в исследовании возрастания и убывания производной функции на реальных вариантах ЕГЭ, постоянно обновляемых с учетом последних изменений и нововведений.

Производной. Если производная функции положительна для любой точки интервала, то функция возрастает, если отрицательна – убывает.

Чтобы найти промежутки возрастания и убывания функции, нужно найти область ее определения, производную, решить неравенства вида F’(x) > 0 и F’(x)

Решение. 4 = 2· (4 — x)/x³.

3. Решим неравенства y’ > 0 и y’ 0;
(4 — x)/x³

4. Левая часть неравенства имеет один действительный х = 4 и обращается в при x = 0. Поэтому значение x = 4 включается и в промежуток , и в промежуток убывания, а точка 0 не включается .
Итак, искомая функция возрастает на промежутке x ∈ (-∞; 0) ∪ .

4. Левая часть неравенства имеет один действительный х = 4 и обращается в при x = 0. Поэтому значение x = 4 включается и в промежуток , и в промежуток убывания, а точка 0 не включается .
Итак, искомая функция возрастает на промежутке x ∈ (-∞; 0) ∪ .

Источники:

  • как найти на функции промежутки убывания

Функция представляет собой строгую зависимость одного числа от другого, или значения функции (y) от аргумента (х). Каждый процесс (не только в математике), может быть описан своей функцией, которая будет иметь характерные особенности: промежутки убывания и возрастания, точки минимумов и максимумов и так далее.

Вам понадобится

  • — бумага;
  • — ручка.

Инструкция

Пример 2.
Найти промежутки убывания f(x)=sinx +x.
Производная данной функции будет равна: f’(x)=cosx+1.
Решая неравенство cosx+1

Интервалом монотонности
функции можно назвать промежуток, в котором функция либо только возрастает, либо только убывает. Ряд определенных действий поможет найти такие диапазоны для функции, что нередко требуется в алгебраических задачах подобного рода.

Инструкция

Первым шагом в решении задачи по определению интервалов, в которых функция монотонно возрастает или убывает, станет вычисление данной функции. Для этого узнайте все значения аргументов (значения по оси абсцисс), для которых можно найти значение функции. Отметьте точки, в которых наблюдаются разрывы. Найдите производную функции. Определив выражение, которое представляет собой производную, приравняйте его к нулю. После этого следует найти корни получившегося . Не про область допустимых .

Точки, в которых функция либо в которых ее производная равна нулю, представляют собой границы интервалов монотонности
. Эти диапазоны, а также точки, их разделяющие, следует последовательно внести в таблицу. Найдите знак производной функции в полученных промежутках. Для этого подставьте в выражение, соответствующее производной, любой аргумент из интервала. Если результат положительный, функция в данном диапазоне возрастает, в обратном случае — убывает. Результаты вносятся в таблицу.

В строку, обозначающую производную функции f’(x), записывается соответствующий значениям аргументов : «+» — если производная положительна,«-» — отрицательна или «0» – равна нулю. В следующей строке отметьте монотонность самого исходного выражения. Стрелка вверх соответствует возрастанию, вниз – убыванию. Отметьте функции. Это точки, в которых производная равна нулю. Экстремум может быть либо точкой максимума, либо точкой минимума. Если предыдущий участок функции возрастал, а текущий убывает, это точка максимума. В случае, когда до данной точки функция убывала, а теперь возрастает – это точка минимума. Внесите в таблицу значения функции в точках экстремума.

Источники:

  • что такое определение монотонность

Исследование поведения функции, имеющей сложную зависимость от аргумента, проводится с помощью производной. По характеру изменения производной можно найти критические точки и участки роста или убывания функции.

Экстремумы функции

Определение 2

Точка $x_0$ называется точкой максимума функции $f(x)$, если существует такая окрестность данной точки, что для всех $x$ из этой окрестность выполняется неравенство $f(x)le f(x_0)$.

Определение 3

Точка $x_0$ называется точкой максимума функции $f(x)$, если существует такая окрестность данной точки, что для всех $x$ из этой окрестность выполняется неравенство $f(x)ge f(x_0)$.

Понятие экстремума функции тесно связано с понятием критической точки функции. Введем её определение.

Определение 4

$x_0$ называется критической точкой функции $f(x)$, если:

1) $x_0$ — внутренняя точка области определения;

2) $f»left(x_0right)=0$ или не существует.

Для понятия экстремума можно сформулировать теоремы о достаточных и необходимых условиях его существования.

Теорема 2

Достаточное условие экстремума

Пусть точка $x_0$ является критической для функции $y=f(x)$ и лежит в интервале $(a,b)$. Пусть на каждом интервале $left(a,x_0right) и (x_0,b)$ производная $f»(x)$ существует и сохраняет постоянный знак. Тогда:

1) Если на интервале $(a,x_0)$ производная $f»left(xright)>0$, а на интервале $(x_0,b)$ производная $f»left(xright)

2) Если на интервале $(a,x_0)$ производная $f»left(xright)0$, то точка $x_0$ — точка минимума для данной функции.

3) Если и на интервале $(a,x_0)$, и на интервале $(x_0,b)$ производная $f»left(xright) >0$ или производная $f»left(xright)

Данная теорема проиллюстрирована на рисунке 1.

Рисунок 1. Достаточное условие существования экстремумов

Примеры экстремумов (Рис. 2).

Рисунок 2. Примеры точек экстремумов

Правило исследования функции на экстремум

2) Найти производную $f»(x)$;

7) Сделать выводы о наличии максимумов и минимумов на каждом промежутке, используя теорему 2.

Возрастание и убывание функции

Введем, для начала, определения возрастающей и убывающей функций.

Определение 5

Функция $y=f(x)$, определенная на промежутке $X$, называется возрастающей, если для любых точек $x_1,x_2in X$ при $x_1

Определение 6

Функция $y=f(x)$, определенная на промежутке $X$, называется убывающей, если для любых точек $x_1,x_2in X$ при $x_1f(x_2)$.

Исследование функции на возрастание и убывание

Исследовать функции на возрастание и убывание можно с помощью производной.

Для того чтобы исследовать функцию на промежутки возрастания и убывания, необходимо сделать следующее:

1) Найти область определения функции $f(x)$;

2) Найти производную $f»(x)$;

3) Найти точки, в которых выполняется равенство $f»left(xright)=0$;

4) Найти точки, в которых $f»(x)$ не существует;

5) Отметить на координатной прямой все найденные точки и область определения данной функции;

6) Определить знак производной $f»(x)$ на каждом получившемся промежутке;

7) Сделать вывод: на промежутках, где $f»left(xright)0$ функция возрастает. 2-30x+36$;

3) $f»left(xright)=0$;

4) $f»(x)$ существует во всех точках области определения;

5) Координатная прямая:

Рисунок 3.

6) Определить знак производной $f»(x)$ на каждом промежутке:

.

Областью значений функции явл. промежуток [ 1; 3].

1. При x = -3, x =- 1, x = 1,5, х=4,5 значение функции равно нулю.

Значение аргумента, при котором значение функции равно нулю, называют нулем функции.

//т.е. для данной функции числа -3;-1;1,5; 4,5 являются нулями.

2. На промежутках [ 4,5; 3) и (1; 1,5) и (4,5;5,5] график функции f расположен над осью абсцисс, а на промежутках (-3; -1) и (1,5; 4,5) под осью абсцисс, это объясняется так -на промежутках [ 4,5; 3) и (1; 1,5) и (4,5;5,5] функция принимает положительные значения, а на промежутках (-3; -1) и (1,5; 4,5) отрицательные.

Каждый из указанных промежутков (там где функция принимает значения одного и того же знака) называют промежутком знакопостоянства функции f. //т.е. например, если взять промежуток (0; 3), то он не является промежутком знакопостоянства данной функции.

В математике принято при поиске промежутков знакопостоянства функции указывать промежутки максимальной длины. //Т.е. промежуток (2; 3) является промежутком знакопостоянства
функции f, но в ответ следует включить промежуток [ 4,5; 3), содержащий промежуток (2; 3).

3. Если перемещаться по оси абсцисс от 4,5 до 2, то можно заметить, что график функции идет вниз, то есть значения функции уменьшаются. //В математике принято говорить, что на промежутке [ 4,5; 2] функция убывает.

С увеличением x от 2 до 0 график функции идет вверх, т.е. значения функции увеличиваются. //В математике принято говорить, что на промежутке [ 2; 0] функция возрастает.

Функцию f называют , если для любых двух значений аргумента x1 и x2 из этого промежутка таких, что x2 > x1, выполняется неравенство f (x2) > f (x1). // или Функцию называют возрастающей на некотором промежутке
, если для любых значений аргумента из этого промежутка большему значению аргумента соответствует большее значение функции. //т.е. чем больше х, тем больше у.

Функцию f называют убывающей на некотором промежутке
, если для любых двух значений аргумента x1 и x2 из этого промежутка таких, что x2 > x1, выполняется неравенство f(x2)убывающей на некотором промежутке, если для любых значений аргумента из этого промежутка большему значению аргумента соответствует меньшее значение функции. //т.е. чем больше х, тем меньше у.

Если функция возрастает на всей области определения, то ее называют возрастающей
.

Если функция убывает на всей области определения, то ее называют убывающей
.

Пример 1.
график возрастающей и убывающей функций соотвественно.

Пример 2.

Определить явл. ли линейная функция f (x) = 3x + 5 возрастающей или убывающей?

Доказательство. Воспрользуемся определениями. Пусть х1 и x2 произвольные значения аргумента, причем x1

Как найти убывающие интервалы с помощью графических функций

Все ресурсы исчисления 1

10 Диагностические тесты
438 практических тестов
Вопрос дня
Карточки
Learn by Concept

← Предыдущая 1 2 3 Следующая →

Исчисление 1 Помощь »
Функции »
Графические функции »
Интервалы »
Уменьшение интервалов »
Как найти убывающие интервалы по графикам функций

Найдите интервал(ы), на которых следующая функция убывает. График, чтобы перепроверить свой ответ.

Возможные ответы:

Никогда

Всегда

Правильный ответ:

4

4

5

4
Объяснение:

Чтобы определить, когда функция убывает, нужно сначала взять производную, затем приравнять ее к 0, а затем найти, между какими нулевыми значениями функция отрицательна.

Сначала возьмем производную:

Приравняем к 0 и решим:

Теперь проверьте значения со всех сторон, чтобы найти, когда функция отрицательна и, следовательно, уменьшается. Я проверю значения -6, 0 и 2.

Поскольку единственное отрицательное значение — это когда x = 0, интервал уменьшается только на интервале, который включает 0. Поэтому , наш ответ:

Сообщить об ошибке График, чтобы перепроверить свой ответ.

Возможные ответы:

Всегда

Никогда

Правильный ответ:

4

4

5

4
Объяснение:

Чтобы определить, когда функция убывает, нужно сначала взять производную, затем приравнять ее к 0, а затем найти, между какими нулевыми значениями функция отрицательна.

Сначала возьмем производную:

Приравняем к 0 и решим:

Теперь проверьте значения со всех сторон, чтобы найти, когда функция отрицательна и, следовательно, уменьшается. Я проверю значения 0, 2 и 10.

Поскольку единственное отрицательное значение — это когда x = 0, интервал уменьшается только на интервале, который включает 2. Следовательно, наш ответ:

Сообщить об ошибке

Увеличивается или уменьшается на интервале?

Возможные ответы:

Функция не возрастает и не убывает на отрезке .

Возрастание, так как первая производная на интервале положительна.

Убывающая, так как первая производная на интервале положительна.

Убывающая, так как первая производная от  отрицательна на функции .

Возрастает, поскольку вторая производная на интервале положительна .

Правильный ответ:

Убывающая, так как первая производная от  отрицательна на функции .

Объяснение:

Чтобы найти возрастающий или убывающий интервал, нам нужно выяснить, положительна или отрицательна первая производная на данном интервале. Итак, найдите , уменьшив каждый показатель степени на единицу и умножив на исходное число.

Далее мы можем найти  и  и посмотреть, являются ли они положительными или отрицательными.

Оба отрицательны, поэтому наклон касательной к  отрицательный, поэтому  убывает.

Сообщить об ошибке

Увеличивается или уменьшается на заданном интервале? Откуда вы знаете?

Возможные ответы:

Возрастание, так как  положительно на интервале .

Уменьшение, так как  отрицательно на интервале .

Уменьшение, так как  отрицательно на интервале .

Недостаточно информации, чтобы определить, увеличивается или уменьшается значение  на интервале .

Возрастание, так как  на интервале положительна.

Правильный ответ:

Возрастание, так как положительно на интервале .

Пояснение:

Напомним, что функция возрастает в точке, если ее первая производная положительна, и убывает, если ее первая производная в этой точке отрицательна. Следовательно, мы должны начать с нахождения f'(x). Однако я начну с объединения одинаковых членов и представления f(x) в стандартной форме:

Затем подключим каждую из наших конечных точек, чтобы увидеть, каков знак f'(x).

Итак, f'(x) положительна на данном интервале, поэтому мы знаем, что f(x) возрастает на данном интервале.

Сообщить об ошибке

Найдите интервалы, в которых следующая функция убывает.

Возможные ответы:

Правильный ответ:

Объяснение:

Первый шаг — найти первую производную.

Мы можем разложить на множители, чтобы получить

.

Теперь нам нужно решить, когда получить критические точки. Обратите внимание, как факторизация 2 немного упростила выражение.

Последний шаг — попробовать точки во всех регионах, чтобы увидеть, какой диапазон дает отрицательное значение для .

Если мы подставим число из первого диапазона в , т. е. получим положительное число.

Из второго диапазона мы получаем отрицательное число.

Из третьего диапазона мы получаем положительное число.

Таким образом, второй диапазон дает нам значения, в которых функция убывает, поскольку  отрицательно в этом диапазоне, поэтому  является ответом.

Сообщить об ошибке

На каком интервале функция, показанная на графике выше, строго убывает?

Возможные ответы:

Интервал C

Интервал B

Интервал A

Интервал E

Интервал D

Правильный ответ:

Интервал E

Объяснение:

Функция строго убывает на интервале, если для любого интервала (т. е. наклон всегда меньше нуля)

Интервал E — единственный интервал, на котором функция проявляет это свойство.

Сообщить об ошибке

Let .

На каком открытом интервале(ах)   уменьшается?

Возможные ответы:

Нет открытых интервалов, на которых функция убывает.

Правильный ответ:

Объяснение:

 убывает на интервалах, где .

Во-первых, дифференцировать.

Затем найдите значения x, для которых производная отрицательна, решив

.

Затем проверьте интервалы.

Протестируйте их, подставив значения x:

Замена -2,

.

На этом интервале функция возрастает, так как производная на этом интервале положительна.

Заменить 0,

.

На этом интервале функция убывает, так как производная на этом интервале отрицательна.

Заместитель 2,

.

На этом интервале функция возрастает, так как производная на этом интервале положительна.

Таким образом, это единственный интервал, на котором функция убывает.

Сообщить об ошибке

На каком интервале функция убывает?

Возможные ответы:

Правильный ответ:

Объяснение:

Функция убывает, когда первая производная отрицательна. Сначала найдем, когда производная равна нулю. Чтобы найти производную, мы применяем правило отношения

.

Следовательно, производная равна нулю в . Чтобы найти, когда он отрицательный, подключите контрольные точки на каждом из трех интервалов, созданных этими нулями.

Например,

.

Следовательно, функция убывает на

.

Сообщить об ошибке

При каких значениях функция убывает?

Возможные ответы:

Эта функция никогда не уменьшается.

 и

и

 и

и

Правильный ответ:

 и

Объяснение:

Чтобы определить, где функция убывает, продифференцируем ее:

Нас интересуют точки, в которых . Чтобы определить эти точки, разложите уравнение на множители:

оно имеет решения в

Это разбивает график на 4 области, и мы можем проверить точки в каждой, чтобы определить, больше или меньше 0. Если меньше нуля, функция убывает.

 отрицательное/убывающее

 положительное/возрастающее

отрицательное/убывающее

положительное/возрастающее

Сообщить об ошибке

Для каких значений функция убывает?

Возможные ответы:

и

и

,

и

Эта функция никогда не уменьшается.

Правильный ответ:

,

Объяснение:

Функция убывает где . Чтобы определить, где это происходит, продифференцируем функцию и найдем, где . Это разделит функцию на интервалы, где она либо возрастает, либо убывает.

Чтобы определить, где это равно нулю, фактор:

это имеет решения для .

Проверить точку в каждой области, чтобы определить, увеличивается она или уменьшается в следующих пределах:

положительный/возрастающий

отрицательный/убывающий

отрицательный/убывающий

положительный/возрастающий

Сообщить об ошибке 0 7 Предыдущий

90 1 2 3 Далее →

Уведомление об авторских правах

Все ресурсы по расчетам 1

10 Диагностические тесты
438 практических тестов
Вопрос дня
Карточки
Учитесь по концепции

Функции возрастания и убывания

Функции возрастания и убывания

Определение возрастания и убывания

Все мы знаем, что если что-то растет, то оно растет, а если
уменьшается он идет вниз. Другой способ сказать, что граф собирается
вверх состоит в том, что его наклон положительный. Если график идет вниз, то
наклон будет отрицательным. Поскольку наклон и производная являются синонимами, мы можем
связывать возрастание и убывание с производной функции. Первый
формальное определение.

Определение возрастания и убывания

Функция увеличение
на интервале если для любого x 1
и х 2 дюймов
интервал затем

х 1 < х 2
подразумевает
f(x 1 )  <  f(x 2 )

Функция уменьшение
на интервале если для любого x 1
и х 2
в промежутке то

х 1 < х 2
подразумевает
f(x 1 ) >  f(x 2 )

Как это относится к деривативам? Напомним, что производная – это
лимит

е(х 2 ) — е(х 1 )

х 2   —  х 1

Если х 1 < х 2 ,
тогда знаменатель будет положительным. Если также  f(x 1 )
< f(x 2 ), то числитель будет положительным, следовательно
производная будет положительной. С другой стороны, если
f(x 1 ) >  f(x 2 ), то числитель будет
отрицательно, и производная будет отрицательной. это приводит нас к
следующая теорема.

Теорема о производных и
Функции увеличения/уменьшения

Пусть f
— дифференцируемая функция на интервале (a,b)
затем

  1. Если ф
    ‘(х) < 0 для х
    в (а, б),
    тогда f есть
    там уменьшается.
  2. Если
    f ‘(x) > 0 для x
    в (а, б),
    тогда f есть
    увеличивается там.
  3. Если
    f ‘(x) = 0 для x
    в (а, б),
    тогда f есть
    постоянный.

Примеры и критические числа

Пример

Определить значения x, где функция

        f(x)
= 2x 3 + 3x 2 — 12x + 7

Раствор

Сначала возьмем производную

        ф ‘(х)
= 6x 2 + 6x — 12

Чтобы определить, где производная положительна, а где отрицательна, найдите
корни. Фактор для получения

        6(x 2
+ х — 2)  =  6(х — 1)(х + 2)

Следовательно, изменение знака может произойти, когда

        х
=  1 и x  =  -2  

Теперь создайте несколько тестовых значений    

 Производная положительна за пределами [-2,1] и
отрицательно внутри [-2,1]. Мы можем заключить
что f возрастает за пределами [-2,1] и
убывает внутри [-2,1]. График
показано ниже.

Мы видели, что значения x, при которых производная равна 0, имеют особое значение.
интерес. Другие точки, где может быть изменение от увеличения до
уменьшение — это когда производная не определена.

Звоним с
критическое число , если
либо f ‘(c) = 0, либо
f'(c) не определено.

Пример

Определите, где указанная ниже функция возрастает, а где убывает.

2
е(х)
знак равно
+   18x
х — 1

2
е(х)
знак равно
+   18x
х — 1

Раствор

Поскольку f(x) не является непрерывной в точке x  =  1, она также не
там дифференцируемый. Следовательно, x = 1 является критической точкой.
Чтобы найти другие критические точки, возьмем производную. Полезно использовать
отрицательные степени вместо дробей здесь.

        f ‘(x)  =  [2(x — 1) -1
+ 18x]’ = -2(x-1) -2 + 18 = 0

        18  =  2(x — 1) -2 Разделить на 2 и умножить на (x — 1) 2

        9(x — 1) 2   =
1    Извлеките квадратный корень из обеих сторон

.

        3(x — 1)  =
1        или     3(x — 1)
= -1

        х  =
4/3    или     x  =  2/3

Это дает нам три критические точки

        х  =
2/3        х  =
1
и        x  =  4/3

Теперь создайте таблицу и определите положительные и отрицательные интервалы

     -2(x — 1) -2 + 18    

х ж'(х)
0 Положительный
. 9 Отрицательный
1,1 Отрицательный
2 Положительный

        Мы можем заключить, что f
увеличивается для значений x меньше 2/3 и значений x больше 4/3.
f уменьшается для значений от 2/3 до 4/3, исключая x =
1. График f показан ниже.


Применение

Вес (в фунтах) новорожденного в течение первых трех месяцев жизни
жизнь может быть смоделирована

        Вт
= 1/3 т 3 + 5/2 т 2 — 19/6 т + 8

, где t измеряется в месяцах. Определите, когда ребенок набрал
веса и когда он похудел.

Раствор

Нас просят найти, когда функция возрастает, а когда
уменьшение. У нас есть

        W’
=  т 2 + 5т — 19/6

Используя формулу квадрата или калькулятор, мы получаем

0,56      или
-5,56

Поскольку домен указан в диапазоне от 0 до 3,
мы используем только 0,56. Теперь постройте таблицу

т Вт'(т)
0 Отрицательный
1 Положительный

Следовательно, функция возрастает при t больше 0,56
и уменьшается при t меньше 0,56

Можно сделать вывод, что младенец терял вес в течение первых 0,56
месяцев своей жизни, а затем начал прибавлять в весе, по крайней мере, до
третий месяц.


Назад
на главную страницу исчисления

Назад к математике
домашняя страница отдела

электронная почта
Вопросы и предложения

Объяснение урока: Увеличение и уменьшение интервалов функции с помощью производных

В этом объяснении мы научимся определять интервалы возрастания и убывания функций, используя первую производную функции.

Производные являются основным инструментом исчисления. Они могут рассказать нам о функции больше, чем просто значение наклона в данной точке или местонахождение критических точек этой функции. Производная имеет приложения в классической механике, например, позволяя нам определять информацию о скорости или ускорении объекта с учетом дифференцируемой функции его смещения.

Прежде чем мы установим, как применять производную для нахождения интервалов возрастания и убывания, давайте начнем с повторения определений возрастающих и убывающих функций.

Определение: возрастание и убывание функций на отрезке

Функция 𝑓 называется возрастающей на отрезке 𝐼, если
𝑓(𝑥)>𝑓(𝑥)𝑥𝑥𝐼.forallin

Функция называется убывающей по 𝐼, если
𝑓(𝑥)𝑓(𝑥)𝑥𝑥𝐼.фораллин

Эти определения должны выполняться для каждые пар точек 𝑥 и 𝑥 в 𝐼 с 𝑥𝑥.

Примечание

Поскольку мы использовали символы «» вместо «≤» и «≥», можно сказать, что функция строго возрастает или убывает по 𝐼. Хотя в этом объяснении мы не будем использовать это соглашение.

Рассмотрим график 𝑦=𝑓(𝑥), показанный ниже.

Функция имеет как возрастающие, так и убывающие интервалы. Мы видим, что график увеличивается (другими словами, имеет наклон вверх) для значений 𝑥−2,5 и 𝑥>1 и убывает (наклон вниз) для значений 𝑥, таких что −2,5𝑥1. Таким образом, функция возрастает на интервалах ]−∞,−2,5[ и ]1,∞[ и убывает на интервале ]−2,5,1[.

Рассматривая определение производной функции, мы можем определить возрастающую и убывающую функции альтернативным способом. Наклон касательной к кривой в данной точке определяется производной функции в этой точке. По этой причине мы можем использовать исчисление, чтобы определить, является ли функция возрастающей или убывающей на 𝐼.

Теорема: возрастание и убывание функций с помощью производных

Рассмотрим функцию 𝑦=𝑓(𝑥), дифференцируемую на ]𝑎,𝑏[.

Если 𝑓′(𝑥)>0 для всех 𝑥∈]𝑎,𝑏[ то 𝑓 возрастает на интервале ]𝑎,𝑏[.

Если 𝑓′(𝑥)0 для всех 𝑥∈]𝑎,𝑏[ то 𝑓 убывает на интервале ]𝑎,𝑏[.

Чтобы доказать эту теорему, положим 𝑥 и 𝑥∈]𝑎,𝑏[, с 𝑥𝑥. Тогда 𝑓(𝑥) дифференцируемо на ]𝑥,𝑥[. По теореме о среднем значении существует число 𝑐∈]𝑥,𝑥[ такое, что
𝑓(𝑥)−𝑓(𝑥)=𝑓′(𝑐)(𝑥−𝑥).

Для первого утверждения, если 𝑓′(𝑥)>0 для 𝑥∈]𝑥,𝑥[ , то 𝑓′(𝑐)>0, так как 𝑐 находится в ]𝑥,𝑥[.

Аналогично, поскольку 𝑥>𝑥, мы можем сказать, что 𝑥−𝑥>0.

Произведение двух положительных чисел положительно, поэтому правая часть уравнения также положительна.

Это означает, что 𝑓(𝑥)−𝑓(𝑥)>0, поэтому 𝑓(𝑥)>𝑓(𝑥).

По определению возрастающих и убывающих функций можно сказать, что если 𝑓′(𝑥)>0 на ]𝑥,𝑥[, то 𝑓 является возрастающей функцией на интервале ]𝑥,𝑥[.

Аналогично можно доказать вторую часть этой теоремы.

Теперь, когда у нас есть теорема, мы можем рассмотреть два примера, связанных с нахождением интервалов возрастания и убывания полиномиальных функций.

Пример 1. Нахождение интервалов возрастания и убывания полиномиальной функции

Определите интервалы, на которых 𝑓(𝑥)=𝑥2−4𝑥+2 увеличивается или уменьшается.

Ответ

Мы знаем, что для дифференцируемой функции 𝑦=𝑓(𝑥) говорят, что 𝑓 возрастает на интервалах, где 𝑓′(𝑥)>0, и убывает на интервалах, где 𝑓′(𝑥)0.

Поскольку 𝑓 многочлен, он везде дифференцируем. Это означает, что мы можем определить интервалы, на которых она увеличивается и уменьшается от ее производной. Используя степенное правило дифференцирования,
𝑓′(𝑥)=4𝑥2−2×4𝑥+0=2𝑥−8𝑥.()

Таким образом, функция будет убывать для значений 𝑥 таких
что 𝑓′(𝑥)0 и возрастает для значений
из 𝑥 такой, что
𝑓′(𝑥)>0.

Чтобы найти такие значения 𝑥, мы начинаем с нахождения значений 𝑥, при которых 𝑓′(𝑥)=0, решая уравнение 2𝑥𝑥−4=0. Это дает нам 𝑥=0, 𝑥=−2 и 𝑥=2. Затем мы определяем знак первой производной на интервалах ]−∞,−2[ ]−2,0[ ]0,2[ и ]2,∞[ подставляя тестовое значение из каждого в выражение 2𝑥 𝑥−4. Мы выберем 𝑥=−3, 𝑥=−1, 𝑥=1 и 𝑥=3.

𝑥 −3 −1 1 3
2𝑥𝑥−4 −30 6 −6 30
Increasing или Уменьшение? По убыванию По возрастанию По убыванию По возрастанию

Функция должна убывать на любых интервалах, где 𝑓′(𝑥)0. Согласно таблице это интервалы ]−∞,−2[ и ]0,2[.

Аналогично, 𝑓(𝑥) увеличивается для таких значений 𝑥, что 𝑓′(𝑥)>0. Это интервалы ]−2,0[ и ]2,∞[.

Функция убывает на интервалах ]−∞,−2[ и ]0,2[ и возрастает на интервалах ]−2,0[ и ]2,∞[.

В нашем первом примере мы увидели, как использование таблицы для расчета тестовых значений для 𝑥, содержащихся в каждом интервале, может помочь нам определить знак производной. Поскольку конечными точками каждого интервала являются значения 𝑥, такие что 𝑓′(𝑥)=0, этот тест говорит нам, что происходит с графиком функции между каждыми двумя стационарными точками. Во втором примере мы будем использовать производную, чтобы найти интервалы возрастания и убывания квадратичной функции.

Пример 2. Нахождение интервалов возрастания и убывания квадратичной функции

Определите интервалы, на которых функция 𝑓(𝑥)=(−3𝑥−12) возрастает и на которых она убывает.

Ответ

Мы знаем, что для функции 𝑦=𝑓(𝑥), дифференцируемой на ]𝑎,𝑏[ верно следующее:

  • Если 𝑓′(𝑥)>0 для всех 𝑥∈]𝑎, 𝑏[ то 𝑓 возрастает на отрезке ]𝑎,𝑏[.
  • Если 𝑓′(𝑥)0 для всех 𝑥∈]𝑎,𝑏[ то 𝑓 убывает на интервале ]𝑎,𝑏[.

Поскольку 𝑓(𝑥) полином, он везде дифференцируем, поэтому мы можем определить интервалы возрастания и убывания, учитывая знак его первой производной. Более того, поскольку 𝑓(𝑥) — дифференцируемая функция, возведенная в степень, мы можем применить общее степенное правило, чтобы найти ее производную.

Это говорит нам о том, что если 𝑦=[𝑔(𝑥)] для дифференцируемой функции 𝑔 и вещественной константы 𝑛, то 𝑦′=𝑛[𝑔(𝑥)]𝑔′(𝑥). Применение этого к 𝑓(𝑥) дает
𝑓′(𝑥)=2(−3𝑥−12)×(−3)=−6(−3𝑥−12)=18𝑥+72.

Функция убывает на любых интервалах, где 𝑓′(𝑥)0. Это дается следующим неравенством:
18𝑥+720𝑥−4.

Аналогично, функция возрастает для таких значений 𝑥, что 𝑓′(𝑥)>0:
18𝑥+72>0𝑥>−4.

Функция убывает на интервале ]−∞,−4[ и возрастает на интервале ]−4,∞[.

Обратите внимание, что существует два математических соглашения о том, что делать с конечными точками (другими словами, включать ли наклон 0 в определение для возрастающих и убывающих функций). Решаем ли мы включать их, во многом зависит от личных предпочтений. В этом объяснении мы будем исключать конечные точки из наших интервалов.

В первых двух примерах вычислялись интервалы возрастания и убывания полиномиальных функций. Важно понимать, что этот процесс также выполняется для неполиномиальных функций, таких как логарифмические функции, экспоненциальные функции, тригонометрические функции и функции, включающие модули, как показано в нашем следующем примере.

Пример 3. Нахождение интервалов возрастания и убывания функции модуля

Определите интервалы, на которых функция 𝑓(𝑥)=(𝑥+3)|𝑥+3| увеличивается и уменьшается.

Ответ

Чтобы установить интервалы возрастания и убывания функции, мы начнем с вычисления ее производной 𝑓′(𝑥). Если 𝑓′(𝑥)>0 на интервале, функция возрастает на этом интервале. Если 𝑓′(𝑥)0 на интервале, функция убывает на этом интервале.

Наша функция является произведением линейной функции и модуля линейной функции, поэтому нам нужно быть осторожными при поиске ее производной. Давайте подумаем о функции |𝑥+3| в виде кусочной функции такой, что
|𝑥+3|=−(𝑥+3)𝑥−3,𝑥+3𝑥≥−3.forfor

Мы можем использовать это определение, чтобы переписать 𝑓(𝑥) следующим образом:
𝑓(𝑥)=−(𝑥+3)𝑥−3,(𝑥+3)𝑥≥−3.для

Тогда, используя общее степенное правило для дифференцирования, получаем
𝑓′(𝑥)=−2(𝑥+3)𝑥−3,2(𝑥+3)𝑥≥−3.forfor

Примечание

Это работает, потому что 𝑓 непрерывно в точке −3, а левая и правая производные оба существуют и равны 0 при 𝑥=−3.

Поскольку 𝑓′(𝑥) положительно для всех действительных 𝑥≠−3, 𝑓 возрастает на ℝ−{−3}. Поскольку 𝑓′(−3)=0, мы также можем включить это значение в интервал возрастания, если захотим. Мы делаем вывод, что 𝑓(𝑥) возрастает по ℝ.

Помимо многочленов и функций, включающих модули, мы можем применить этот процесс к логарифмическим функциям. Мы продемонстрируем это в нашем следующем примере.

Пример 4. Нахождение интервалов, в которых функции, включающие логарифмические функции, возрастают и убывают

Учитывая, что 𝑓(𝑥)=5𝑥−3𝑥−𝑥ln, найдите интервалы, на которых 𝑓 увеличивается или уменьшается.

Ответ

Помните, если 𝑓 дифференцируемо на открытом интервале, то 𝑓 будет возрастать на интервалах, где 𝑓′(𝑥)>0, и убывать на интервалах, где 𝑓′(𝑥)0. Итак, мы начнем с нахождения 𝑓′(𝑥), наблюдая, как домен 𝑓(𝑥) является положительными действительными числами:
𝑓′(𝑥)=10𝑥−3−1𝑥.

Чтобы определить знак этой функции, начнем с нахождения значения ее 𝑥-перехватов, решив 𝑓′(𝑥)=0:
10𝑥−3−1𝑥=010𝑥−3𝑥−1=0(5𝑥+1)(2𝑥−1)=0𝑥=−15𝑥=12.или

Поскольку −15 находится вне области определения 𝑓(𝑥), мы теперь будет определять знак 𝑓′(𝑥) на интервалах 0,12 и 12,∞. Для этого подставляем тестовые значения из каждого интервала в выражение для 𝑓′(𝑥). Выберем 𝑥=110 и 𝑥=1.

𝑥 110 1
10𝑥 — 3–1𝑥 −12 6
Увеличение или уменьшение Уменьшение ,

с со времен . Точно так же он увеличивается при 𝑥=1.

𝑓 возрастает на интервале 12,∞ и убывает на интервале 0,12.

Теперь давайте продемонстрируем этот процесс с помощью функции, являющейся произведением полинома и экспоненты.

Пример 5. Нахождение интервалов возрастания и убывания функции с помощью правила произведения с экспоненциальными функциями

Пусть 𝑓(𝑥)=3𝑥𝑒. Определите промежутки возрастания и убывания этой функции.

Ответ

Мы знаем, что для дифференцируемой функции 𝑦=𝑓(𝑥) 𝑓 будет возрастать на интервалах, где 𝑓′(𝑥)>0, и убывать на интервалах, где 𝑓′(𝑥)0.

Начнем с проверки дифференцируемости функции 𝑓(𝑥). Мы знаем, что произведение двух дифференцируемых функций также дифференцируемо. 3𝑥 является полиномиальной функцией и, следовательно, дифференцируема в своей области определения. Точно так же экспоненциальная функция 𝑒 для вещественных констант 𝑎 также дифференцируема в своей области определения. Это означает, что 𝑓(𝑥) действительно дифференцируема, поэтому мы можем определить интервалы возрастания и убывания по ее производной.

Правило произведения говорит нам, что для двух дифференцируемых функций 𝑢 и 𝑣
дддддд𝑥(𝑢𝑣)=𝑢𝑣𝑥+𝑣𝑢𝑥.

Пусть 𝑢=3𝑥 и 𝑣=𝑒 так, что
ddanddd𝑢𝑥=12𝑥𝑣𝑥=−4𝑒.

Подстановка этих значений в формулу правила произведения дает
𝑓′(𝑥)=3𝑥×−4𝑒+𝑒×12𝑥=−12𝑥𝑒+12𝑥𝑒=12𝑥𝑒(−𝑥+1).

Таким образом, функция будет убывать для значений 𝑥 на интервалах, где 12𝑥𝑒(−𝑥+1)0, и возрастать на интервалах, где 12𝑥𝑒(−𝑥+1)>0. Поскольку 𝑒>0 для всех значений 𝑥, интервалы возрастания и убывания будут полностью зависеть от знака 12𝑥(−𝑥+1).

Давайте вычислим стационарные точки 𝑓(𝑥), решив 12𝑥(−𝑥+1)=0. Это дает нам 𝑥=0 и 𝑥=1. Поэтому подставим значения из интервалов ]−∞,0[ ]0,1[ и ]1,∞[ в выражение 12𝑥(−𝑥+1) и установим их знак. Мы выбираем 𝑥=−1, 𝑥=0,5 и 𝑥=2.

𝑥 −1 0.5 2
12𝑥(−𝑥+1) −24 0.75 −96
Increasing or Decreasing? Уменьшение Возрастание Уменьшение

𝑓 увеличивается, когда 𝑓′(𝑥)>0, и уменьшается, когда 𝑓′(𝑥)0. Следовательно, 𝑓 возрастает на интервале ]0,1[ и убывает на интервалах ]−∞,0[ и ]1,∞[.

В предыдущих примерах мы рассмотрели функции, дифференцируемые во всей области определения. Так будет не всегда, поэтому нам нужно будет тщательно рассматривать область определения функций при нахождении интервалов возрастания и убывания.

Пример 6. Нахождение интервалов, на которых функция, включающая корневую функцию, возрастает и убывает

Найдите интервалы, на которых функция 𝑓(𝑥)=5𝑥√−5𝑥+3 возрастает и убывает.

Ответ

Чтобы установить интервалы возрастания и убывания функции, мы можем рассмотреть ее производную 𝑓′(𝑥). Если 𝑓 дифференцируемо на открытом интервале, то 𝑓 будет возрастать на интервалах, где 𝑓′(𝑥)>0, и убывать на интервалах, где 𝑓′(𝑥)0.

Начнем с проверки дифференцируемости функции 𝑓(𝑥). Мы знаем, что произведение двух дифференцируемых функций также дифференцируемо. 5𝑥 является многочленом и, следовательно, должен быть дифференцируемым для всех действительных чисел, хотя √−5𝑥+3 немного сложнее. Его можно отличить, применив общее правило мощности; однако нам нужно убедиться, что любые значения внутри квадратного корня неотрицательны:
−5𝑥+3≥05𝑥≤3𝑥≤35.

Следует отметить, что технически 𝑓 не дифференцируема при 𝑥=35, так как мы не можем взять правый предел для ее производной в этой точке.

Функция может возрастать и убывать только в своей области определения, поэтому мы будем рассматривать только значения 𝑥≤35.

Чтобы найти выражение для 𝑓′(𝑥), мы можем применить правило произведения, которое говорит нам, что для двух дифференцируемых функций 𝑢 и 𝑣
дддддд𝑥(𝑢𝑣)=𝑢𝑣𝑥+𝑣𝑢𝑥.

Пусть 𝑢=5𝑥 так, что dd𝑢𝑥=5.

Аналогично, пусть 𝑣=√−5𝑥+3=(−5𝑥+3). Затем мы применяем общее степенное правило, чтобы найти его производную.

Это говорит нам о том, что если 𝑦=[𝑔(𝑥)] для дифференцируемой функции 𝑔(𝑥), то 𝑦′=𝑛[𝑔(𝑥)]𝑔′(𝑥), где 𝑛 — вещественная постоянный.

Итак,
dd𝑣𝑥=12(−5𝑥+3)×(−5)=−52(−5𝑥+3).

По правилу произведения,
𝑓′(𝑥)=5𝑥×−52(−5𝑥+3)+(−5𝑥+3)×5=−252𝑥(−5𝑥+3)+5(−5𝑥+3).  

Мы можем упростить работу с этим выражением, создав общий знаменатель 2(−5𝑥+3) и добавив выражения:
𝑓′(𝑥)=-25𝑥2(-5𝑥+3)+5(-5𝑥+3)×2(-5𝑥+3)2(-5𝑥+3)=-25𝑥+10(-5𝑥+3)2( −5𝑥+3)=−75𝑥+302(−5𝑥+3)𝑥35.для

Таким образом, функция будет убывать, когда −75𝑥+302(−5𝑥+3) 0 и увеличивается, когда −75𝑥+302(−5𝑥+3)>0 для значений 𝑥35.

Знаменатель дроби положителен для 𝑥35, поэтому знак производной полностью определяется знаком −75𝑥+30. Следовательно, наклон будет отрицательным для всех значений 𝑥 в области 𝑓′, где выполняется следующее неравенство:
−75𝑥+30075𝑥>30𝑥>25.

Это говорит нам о том, что функция убывает на 25𝑥35.

Аналогично, функция будет возрастать, когда −75𝑥+30>0 и 𝑥35:
−75𝑥+30>075𝑥3𝑥25.

Функция возрастает на −∞,25 и убывает на 25,35.

Как мы видели в предыдущем примере, нужно соблюдать осторожность при дифференциации функций, включающих корень. Это также верно для рациональных функций, где нам нужно убедиться, что знаменатель функции не равен нулю. Будут случаи, когда знаменатель не может быть равен нулю ни для каких реальных значений переменной, и в этом случае мы можем выполнять вычисления, не беспокоясь, как мы увидим в следующем примере.

Пример 7. Нахождение интервалов возрастания и убывания рациональной функции

Определите промежутки, на которых функция 𝑓(𝑥)=7𝑥𝑥+9 возрастает и где она убывает.

Ответ

Чтобы установить интервалы возрастания и убывания функции, мы можем рассмотреть ее производную 𝑓′(𝑥). Если 𝑓 дифференцируемо на открытом интервале, то 𝑓 возрастает на интервалах, где 𝑓′(𝑥)>0, и убывает на интервалах, где 𝑓′(𝑥)0.

Функция 𝑓(𝑥) является частным двух дифференцируемых функций, поэтому она дифференцируема во всей своей области определения. Заметим, что нет реальных значений 𝑥, которые составляют знаменатель, 𝑥+9, равный нулю, поэтому домен равен ℝ.

Чтобы найти производную, мы используем правило отношения, которое гласит, что для дифференцируемых функций 𝑢 и 𝑣,
dd(𝑢𝑣)𝑥=𝑣−𝑢𝑣. dddd

Мы определяем 𝑢=7𝑥, поэтому dd𝑢𝑥=7. Точно так же 𝑣=𝑥+9, поэтому dd𝑣𝑥=2𝑥. Следовательно,
𝑓′(𝑥)=7𝑥+9−7𝑥(2𝑥)(𝑥+9)=−7𝑥+63(𝑥+9).

Теперь мы можем заметить, что знаменатель функция 𝑓′(𝑥) равна 𝑥+9, что положительно для всех действительных значений 𝑥. Это означает, что знак 𝑓′(𝑥) полностью зависит от знака числителя.

Чтобы определить знак −7𝑥+63, решаем −7𝑥+63=0 и затем находим знак тестовых значений в интервалах вокруг этих точек:
−7𝑥+63=07𝑥=63𝑥=9𝑥=±3.

Теперь выберем тестовые значения 𝑥 из интервалов ]−∞,−3[ ]−3,3[ и ]3,∞ [ и определить знак −7𝑥+63 с этими значениями, помня, что функция возрастает на интервалах 𝑥, где 𝑓′(𝑥)>0, и убывает, где 𝑓′(𝑥)0. Выберем 𝑥=−4, 𝑥=0 и 𝑥=4.

7 Возрастание или убывание?

𝑥 −4 0 4
−7𝑥+63 −49 63 −49
Убывающая Возрастающая Убывающая

Следовательно, функция убывает на интервалах ]−∞,−3[ и ]3,∞[ и возрастает на ]−3,3[.

В нашем последнем примере мы покажем, как применять процесс для нахождения интервалов возрастания и убывания тригонометрических функций.

Пример 8. Нахождение интервалов возрастания и убывания функции, включающей тригонометрические функции

Для 0𝑥2𝜋5 найдите интервалы, на которых 𝑓(𝑥)=5𝑥+35𝑥coscos увеличивается или уменьшается.

Ответ

Мы можем определить интервалы возрастания или убывания дифференцируемой функции, проверяя знак ее первой производной.

Поскольку 𝑓(𝑥) является суммой двух функций косинуса, а функция косинуса дифференцируема по ℝ, мы можем заключить, что 𝑓(𝑥) дифференцируема по ℝ.

Мы знаем, что для вещественных констант 𝑎 ddcossin𝑥𝑎𝑥=−𝑎𝑎𝑥. Поэтому мы можем использовать комбинацию этой формулы и общего правила мощности, чтобы дифференцировать каждую часть функции:
𝑓′(𝑥)=2((5𝑥))×(−5(5𝑥))+5(−3(5𝑥))=−10(5𝑥)(5𝑥)−15(5𝑥)=−5(5𝑥)( 2(5𝑥)+3).cossinsincossinsinsincos

Таким образом, функция будет убывать на интервалах, где −5(5𝑥)(2(5𝑥)+3)0sincos, и возрастать на интервалах, где −5(5𝑥)(2( 5𝑥)+3)>0sincos.

Это означает, что нам нужно определить знак синкосов −5(5𝑥)(2(5𝑥)+3) в различных точках. Для этого найдем 𝑥-пересечения графика 𝑓(𝑥)=−5(5𝑥)(2(5𝑥)+3)sincos, решив −5(5𝑥)(2(5𝑥)+3)= 0синкос.

Это происходит, когда −5(5𝑥)=0sin или 2(5𝑥)+3=0cos.

Итак,
−5 (5𝑥) = 0 (5𝑥) = 05𝑥 = 0+2𝜋𝑛𝜋+2𝜋𝑛𝑛∈ℤ𝑥 = 0+25𝜋𝑛𝜋5+25𝜋𝑛𝑛∈ℤ.SinsinorForor

В интервале 0𝑥2𝜋5, единственное решение — когда 𝑛 = 0: 𝑥 = 𝜋5.

Процесс решения 2(5𝑥)+3=0cos выглядит следующим образом:
2(5𝑥)+3=02(5𝑥)=−3(5𝑥)=−32.coscoscos

У этого уравнения нет реальных решений, поэтому единственный 𝑥-отрезок графика 𝑓(𝑥)=−5 (5𝑥)(2(5𝑥)+3)синкос равен 𝑥=𝜋5. Поэтому мы выберем тестовые значения из интервалов 0,𝜋5 и 𝜋5,2𝜋5 и проверим знак производной в этих точках. Выберем 𝑥=𝜋10 и 𝑥=3𝜋10.

𝑥 𝜋10 3𝜋10
−5(5𝑥)(2(5𝑥)+3)sincos −15 15
Increasing or Decreasing? Убывающая Возрастающая

Поскольку производная отрицательна на интервале 0,𝜋5 и положительна на интервале 𝜋5,2𝜋5, функция убывает на 0,𝜋5, и возрастает на  2𝜋5.

Понравилась статья? Поделить с друзьями:
  • Как найти папку если она пропала
  • Рассказ как найти общий язык
  • Как найти человеко дни за год
  • Грамоты благодарности как составить
  • Как найти резервное копирование в вайбере